Sei sulla pagina 1di 78

Fluids and Electrolytes

A 74-year-old man with low ejection fraction systolic heart failure comes to the Emergency Department with shortness of breath. His past
medical history is significant for small cell lung cancer and he is on metformin. On physical exam, the patient is found to have 2+ pitting edema
in both lower extremities. The patient is started on furosemide. His metabolic panel is within normal limits with the exception of serum sodium of
128 mEq/L.
Syndrome of inappropriate antidiuretic hormone production (SIADH) is characterized by which of the following?
1)
2)
3)
4)
5)

hyponatremia/low serum osmolality/hyperkalemia


hypernatremia/high serum osmolality/hypokalemia
hyponatremia/normal urinary osmolality
hypokalemia/high urinary osmolality
hyperkalemia/low urinary osmolality

The correct answer is choice C

The patients euvolemic status and hyponatremia suggests that SIADH is the most cause of of his hyponatremia. Syndrome of inappropriate
antidiuretic hormoneis paraneoplastic syndrome commonly associated with small cell carcinoma of the lungs but the hyponatremia is euvolemic.
Patients would not present with physical findings of volume overload (i.e. ascites, pitting edema).

Hyponatremia is an electrolyte disturbance in which the sodium concentration in the serum is lower than normal. Sodium is the dominant
extracellular cation and cannot freely cross the cell membrane. Normal serum sodium levels are between 135-145 mEq/L. Hyponatremia is
defined as a serum level of less than 135 mEq/L and is considered severe when the serum level is below 125 mEq/L.

Hyponatremia is most often a complication of other medical illnesses in which either fluids rich in sodium are lost (diarrhea or vomiting) or
excess water accumulates in the body at a higher rate than can be excreted (for example in congestive heart failure, syndrome of inappropriate
antidiuretic hormone, SIADH, or polydipsia). Regarding sodium loss as a cause of hyponatremia, it is important to note that such losses promote
hyponatremia in only an indirect manner. In particular, hyponatremia occurring in association with sodium loss does not reflect inadequate
sodium availability as a result of the losses. Rather, the sodium loss leads to a state of volume depletion, with volume depletion serving as
signal for the release of ADH (anti-diuretic hormone). As a result of ADH-stimulated water retention, blood sodium becomes diluted and
hyponatremia results.

Symptoms of hyponatremia include nausea and vomiting, headache, confusion, lethargy, fatigue, appetite loss, restlessness and irritability,
muscle weakness, spasms, or cramps, seizures, and decreased consciousness or coma. The presence and severity of symptoms are
associated with the level of serum sodium, with the lowest levels of serum sodium associated with the more prominent and serious symptoms.

Neurological symptoms often show for extremely low levels of sodium. When sodium levels in blood become too low, excess water enters cells
and causes the cells to swell. Swelling in the brain is especially dangerous because the brain is confined by the skull and is unable to expand.
Neurological symptoms most often are due to very low serum sodium levels (usually <115 mEq/L), resulting in intracerebral osmotic fluid shifts
and brain edema. This neurological symptom complex can lead to tentorial herniation with subsequent brain stem compression and respiratory
arrest, resulting in death in the most severe cases.

The imbalance between sodium and water in blood may occur in three primary ways and is based on the patients volume status:

Hypervolemic hyponatremia, excess water dilutes the sodium concentration, causing low sodium levels. Hypervolemic hyponatremia is
commonly the result of kidney failure, heart failure or liver failure.

Euvolemic hyponatremia, normal water levels are combined with low sodium levels. This condition is commonly due to chronic health
conditions, cancer or certain medications.

Hypovolemic hyponatremia, water and sodium levels are both low. This may occur, for example, when exercising in the heat without
replenishing fluid electrolytes or with marked blood loss.

hyponatremia/low serum osmolality/hyperkalemia (choice A) is incorrect as SIADH has no effect or serum potassium or volume status.

hypernatremia/high serum osmolality/hypokalemia (choice B) is incorrect as SIADH has no effect or serum potassium or volume status.

hypokalemia/high urinary osmolali (choice D) is incorrect as SIADH has no effect or serum potassium or votylume status.

hyperkalemia/low urinary osmolality (choice E) is incorrect as SIADH has no effect or serum potassium or volume status.

A 76 year old man comes to the ER from a nursing home with altered mental status after hitting his head from a fall. A cat scan of the head is
obtained. According to his nursing aide, he has been drinking and eating less the past month. He has a history of receiving chemotherapy for
small cell lung cancer and has been receiving a large amount of IV fluids to prevent tumor lysis syndrome. He also has a history of uncontrolled
diabetes with a hemoglobin A1C of 10%. A finger stick reveals a blood glucose of 432. His metabolic panel is within normal limits with the
exception of serum sodium of 155 mEq/L and a urine specific gravity of 1.004.
All of the following are possible factors contributing to the patients hypernatremia except?
1)
2)
3)
4)
5)

Syndrome of inappropriate antidiuretic hormone


marked hyperglycemia
excessive IV fluids
severe water deprivation
diabetes insipidus

The correct answer is choice A

Syndrome of inappropriate antidiuretic hormone is a paraneoplastic syndrome commonly associated with small cell carcinoma of the lungs that
results in a euvolemic hyponatremia. A paraneoplastic syndrome is a disease or symptom that is the consequence of the presence of cancer in
the body, but is not due to the local presence of cancer cells. These phenomena are mediated by humoral factors (by hormones or cytokines)
excreted by tumor cells or by an immune response against the tumor. Paraneoplastic syndromes are typical among middle aged to older
patients (as are all cancers), and they most commonly present with cancers of the lung, breast, ovaries or lymphatic system (a lymphoma).
Sometimes the symptoms of paraneoplastic syndromes show even before the diagnosis of a malignancy.

Marked hyperglycemia (chocie B) can cause a hypolvolemic hypernatremia from the excessive loss of water from the urinary tract due to
osmotic diuretics of excessive blood glucose.

Excessive IV fluids (choice C) can be an iatrogenic cause of hypernatremia. Typically, the IV fluid is hypertonic fluid (a fluid with a higher
concentration of solutes than the remainder of the body) and the hypernatremia results from vigorous resuscitation .

Severe water deprivation (choice D) would cause a hypovolemic hypernatremia. Inadequate intake of water, typically in elderly or otherwise
disabled patients who are unable to take in water as their thirst dictates is common in nursing homes and the most common cause of
hypernatremia.

Diabetes insipidus (choice E) is characterized by increased thirst (polydipsia) and increased urination (polyuria) with the inability to to
concentrate urine due to a lack of ADH. Therefore, urine specific gravity will be low (<1.006) and serum osmolality will be >250. Nephrogenic
diabetes insipidus (lack of response to ADH) can be distinguished from central (loss of ADH production) because nephrogenic DI will not
respond to DDAVP which is a ADH analog. This patient is suffering from central diabetes insipidus due to the traumatic head injury.

A 37 year old African-American woman presents to her primary care doctor complaining of vision problems, fatigue, and constipation for the
past six months. She has a history of bipolar disorder, sarcoidosis, high blood pressure, and malabsorption for which she takes ithium,
prednisone, and furosemide respectively. Her physical exam is unremarkable except for tender red skin nodules. Serum chemistry is
remarkable for elevated calcium at 11.6, low phosphate at 2.1, slightly elevated alkaline phosphatase at 98, and elevated 1-alpha hydroxylase.
All of the following are causes of hypercalcemia, except:
1)
2)
3)
4)
5)

Primary hyperparathyroidism
Lithium therapy
Granulomatous Disease
Furosemide therapy
All of the above are true

The correct answer is choice D

Hypercalcaemia is an elevated calcium level in the blood (Normal range: 9-10.5 mg/dL). It can be an asymptomatic laboratory finding.
However, an elevated calcium level is often indicative of other diseases and a differential diagnosis should be undertaken if it persists. It can be
due to excessive skeletal calcium release, increased intestinal calcium absorption, or decreased renal calcium excretion.

EKG findings of a shortened QT interval and a widened T wave suggest hypercalcaemia and can lead to abnormal heart rhythms. Finally, peptic
ulcers may also occur. Symptoms are more common at high calcium blood values (12.0 mg/dLl). Severe hypercalcaemia (above 1516 mg/dL)
is considered a medical emergency: at these levels, coma and cardiac arrest can result. Initial therapy consists of IV fluids and diuretics
(furosemide) supplemented with bisphosphonates and calcitonin.

The general mnemonic for remembering the physical symptoms of hypercalcaemia: "groans (constipation), moans (psychotic noise), bones
(bone pain, especially if PTH is elevated), stones (kidney stones), and psychiatric overtones (including depression and confusion)." Other
symptoms can include fatigue, anorexia, nausea, vomiting, pancreatitis and increased urination.

Intestinal malabsorption can lead to hypocalcemia due to decreased absoprtion of calcium in the gut.

Primary hyperparathyroidism (choice A) causes hypercalcemia through the excessive secretion of parathyroid hormone (PTH), usually by an
adenoma (benign tumors) of the parathyroid glands. Primary hyperparathyroidism is also a feature of several familial endocrine disorders:
Multiple endocrine neoplasia type 1 and type 2A (MEN type 1 and MEN type 2A), and familial hyperparathyroidism.

Lithium therapy (choice B) is a commonly used mood stabilizer that can lead to electrolyte disturbances such as hypercalcemia. Other toxic
effects include tremor, ataxia, dysarthria, nystagmus, renal impairment, and convulsions.

Granulomatous Disease (choice C) such as sarcoidosis is a well know cause of hypercalcemia. The granulomas have elevated levels of 1-alpha
hydroxylase, an enzyme normally found in the renal proximal tubules that is responsible for the final step in biosynthesis of Vitamin D3. Elevated
levels of Vitamin D cause hypercalcemia by increasing calciums absorption in the gut and decreasing its excretion by the kidneys.

Furosemide (choice D) has a well known side effect of hypocalcemia and can be remembered with mnemonic the loops lose calcium.

Proper balance of electrolytes is essential for various normal body functions such as muscle coordination, heart function, fluid absorption and
excretion, nerve function, and concentration. Body fluid volume and electrolyte concentration are normally maintained within very narrow limits
despite wide variations in dietary intake, metabolic activity, and environmental stresses. Homeostasis of body fluids is preserved primarily by the

kidneys. Disturbances in this homeostasis can have a varied manifestation ranging from benign to critical.
Which of the following is the most common electrolyte abnormality observed in a general hospitalized population?
1)
2)
3)
4)
5)

Hyperkalemia
Hyponatremia
Hypomagnesemia
Hyperphospatemia
Hypercalcemia

The correct answer is choice B.

Hyponatremia is the most common electrolyte disorder in generalized hospital populations with about 15-20% hospital patients suffering from it.
Hyponatremia has been reported in up to 30% of elderly patients in nursing homes and is also present in approximately 30% of depressed
patients on selective serotonin reuptake inhibitors. Hyperkalemia has been diagnosed in up to 8% of hospitalized patients in the U.S while
hypermagnesemia has a frequency of 10-20% in hospitalized patients. The following chart describes the causes and symptoms of common
electrolyte disturbances.

A 63-year-old alcoholic man presents to the Emergency Department with increasing abdominal girth and right upper quadrant pain. He has a
history of admissions for congestive heart failure exacerbations, for which he takes diuretics and digitalis. He also has a history of G6PD and
was taking gentamicin a week ago. On physical exam, the patient has a narrow pulse pressure and an S3 and S4 are heard on auscultation of
the heart. Examination of the right neck veins reveals venous pulsations that are best seen when the head of the bed is elevated to ninety
degrees. The abdomen is distended with shifting dullness.
Which of the following conditions is NOT associated with hypomagnesemia?
1)
2)
3)
4)

Alcoholism
Antibiotics
Digitalis
Diuretics

5) Hemolysis
The correct answer is choice E

Hypomagnesemia is an electrolyte disturbance in which there is an abnormally low level of magnesium in the blood. Usually a serum level less
than 0.7 mmol/L is used as reference. It may result from a number of conditions including inadequate intake of magnesium, chronic diarrhea,
malabsorption, alcoholism, chronic stress, diuretic use and other disorders. Low levels of magnesium in blood may mean either there is not
enough magnesium in the diet, the intestines are not absorbing enough magnesium, or the kidneys are excreting too much magnesium.

Deficiency of magnesium causes weakness, muscle cramps, cardiac arrhythmia, increased irritability of the nervous system with tremors,
athetosis, jerking, nystagmus and an extensor plantar reflex. In addition, there may be confusion, disorientation, hallucinations, depression,
epileptic fits, hypertension, tachycardia and tetany. Patients with refractory hypokalemia also have concomitant hypomagnesemia, not
responding to potassium supplementation. Magnesium is needed for the ATPase, Na-K-pump.

Magnesium concentration in erythrocytes is approximately three times greater than in serum, therefore hemolysis can increase plasma
magnesium. Hypermagnesemia is expected only in massive hemolysis.

Alcoholism (choice A) resules in hypomagnesemia due to malnutrition and chronic diarrhea. Alcohol stimulates renal excretion of magnesium,
which is also increased because of alcoholic and diabetic ketoacidosis, hypophosphatemia and hyperaldosteronism resulting from liver disease.
Also, hypomagnesemia is related to thiamine deficiency because magnesium is needed for transforming thiamine into thiamine pyrophosphate.

Antibiotics (choice B) (i.e. aminoglycoside, amphotericin, pentamidine, gentamicin, tobramycin, viomycin) block resorption in the loop of Henle.
Patients using these antibiotics often have hypomagnesemia,

Digitalis (choice C) displaces magnesium into the cell. Digitalis causes an increased intracellular concentration of sodium, which in turn
increases intracellular calcium by passively decreasing the action of the sodium-calcium exchanger in the sarcolemma. The increased
intracellular calcium gives a positive inotropic effect.

Loop and thiazide diuretic use (choice D) is the most common cause of hypomagnesemia in the inpatient setting.

A 55-year-old retired salesperson with end stage renal disease presents in clinic complaining of malaise, palpitations and muscle weakness. He
is on anirregular hemodialysis program. A stat EKG is performed showing peaked T waves and a widened QRS complex. His serum potassium
comes back at 6.9 meq/L.
The following are true regarding the treatment of hyperkalemia, except?
1)
2)
3)
4)
5)

Bicarbonate infusion reduces serum potassium by 2-3.5 meq/L


Glucose and insulin infusion policy reduces serum potassium by 0.6-1.2 meq/L
Calcium gluconate infusion does not reduce serum potassium
Potassium exchange resins with sorbitol
Beta agonists' infusion may be additive or alternative to glucose and insulin policy

The correct answer is choice A

Hyperkalemia refers to the condition in which the concentration of the electrolyte potassium (K+) in the blood is elevated. Extreme hyperkalemia
is a medical emergency due to the risk of potentially fatal abnormal heart rhythms (arrhythmia). Normal serum potassium levels are between 3.5
to 5.0 mEq/L; at least 95% of the body's potassium is found inside cells, with the remainder in the blood. This concentration gradient is
maintained principally by the Na+/K+ pump. Increased extracellular potassium levels result in depolarization of the membrane potentials of cells.
This depolarization opens some voltage-gated sodium channels, but not enough to generate an action potential. After a short while, the open
sodium channels inactivate and become refractory, increasing the threshold needed to generate an action potential. This leads to the
impairment of neuromuscular, cardiac, and gastrointestinal organ systems.

The renal elimination of potassium is passive (through the glomeruli), and reabsorption is active in the proximal tubule and the ascending limb of
the loop of Henle. There is active excretion of potassium in the distal tubule and the collecting duct; both are controlled by aldosterone.
Hyperkalemia develops when there is excessive production (oral intake, tissue breakdown) or ineffective elimination of potassium. Ineffective
elimination can be hormonal (in aldosterone deficiency) or due to causes in the renal parenchyma that impair excretion.

Symptoms are fairly nonspecific and generally include malaise, palpitations and muscle weakness; mild hyperventilation may indicate a
compensatory response to metabolic acidosis, which is one of the possible causes of hyperkalemia. Often, however, the problem is detected
during screening blood tests for a medical disorder, or it only comes to medical attention after complications have developed, such as cardiac

arrhythmia or sudden death. During the medical history taking, a physician will focus on kidney disease and medication use.

When arrhythmias occur, or when potassium levels exceed 6.5 mmol/l, emergency lowering of potassium levels is mandated. Several agents
are used to lower K levels. Choice depends on the degree and cause of the hyperkalemia, and other aspects of the patient's condition.
Bicarbonate therapy is effective in cases of metabolic acidosis. The bicarbonate ion will stimulate an exchange of cellular H+ for Na+, thus
leading to stimulation of the sodium-potassium ATPase.

Glucose and insulin infusion policy reduces serum potassium by 0.6-1.2 meq/L (choice B) by leading to a shift of potassium ions into cells,
secondary to increased activity of the sodium-potassium ATPase. Its effects last a few hours, so it sometimes needs to be repeated while other
measures are taken to suppress potassium levels more permanently.

Calcium gluconate infusion does not reduce serum potassium (choice C). Calcium supplementation (calcium gluconate 10%, preferably
through a central venous catheter as the calcium may cause phlebitis) does not lower potassium but decreases myocardial excitability,
protecting against life threatening arrhythmias. It tends to be used if there are characteristic EKG changes.

potassium exchange resins with sorbitol (choice D) such as Kayexalate promotes the lowering of potassium through the GI tract. The potassium
is removed when the patient has a bowel movement.

Beta agonists' infusion may be additive or alternative to glucose and insulin policy (choice E). Albuterol is a 2-selective catecholamine that is
administered by nebulizer (e.g. 1020 mg). This drug promotes movement of K into cells, lowering the blood levels.

An 80 year-old male presents with lethargy and anorexia for three days duration. He has a history of atrial fibrillation and has been rate
controlled for several years with digoxin. His history is otherwise unremarkable, and his physical exam is within normal limits. After labs are
drawn he is found to have a plasma potassium level of 6.0 mEq/L and a digoxin level of 4.0 (normal therapeutic levels are between 0.8 - 2.0).
Each of the following may be a direct consequence of severe magnesium deficiency except:
1) digitalis-induced arrhythmias
2) hypocalcemia
3) hypokalemia

4) hyponatremia
5) confusion
The correct answer is choice D

Magnesium is needed for the adequate function of the Na+/K+-ATPase pumps in the cells of the heart. A lack of it depolarises and results in
tachyarrhythmia. Magnesium intravenously helps in refractory arrhythmia, most notably torsade de pointes. Others are ventricular tachycardia,
supraventricular tachycardia and atrial fibrillation. The effect is based upon decreased excitability by depolarisation and the slowing down of
electric signals in the AV-node. Magnesium is a negative inotrope as a result of decrease calcium influx and calcium release from intracellular
storage. It is just as effective as verapamil. In myocardial infarction there is a functional lack of magnesium, supplementation will decrease
mortality.

Deficiency of magnesium causes weakness, muscle cramps, cardiac arrhythmia, increased irritability of the nervous system with tremors,
athetosis, jerking, nystagmus and an extensor plantar reflex. In addition, there may be confusion, disorientation, hallucinations, depression,
epileptic fits, hypertension, tachycardia and tetany.

Magnesium deficiency is not uncommon in hospitalized patients. Elevated levels of magnesium. 10-20% of all hospital patients and 60-65% of
patient in the intensive care unit have hypomagnesemia. Hypomagnesemia is underdiagnosed, as testing for serum magnesium levels is not
routine. Hypomagnesemia results in increased mortality. Treatment of hypomagnesemia depends on the degree of deficiency and the clinical
effects. Oral replacement is appropriate for patients with mild symptoms, while intravenous replacement is indicated for patients with severe
clinical effects.

Magnesium deficiency has no effect on serum sodium levels.

Digitalis-induced Arrhythmias (choice A) can result from magnesium deficiency. Magnesium is needed for the adequate function of the Na+/K+ATPase pumps in the cells of the heart. A lack of it depolarises and results in tachyarrhythmia. Magnesium inhibits release of potassium, a lack
of magnesium increases loss of potassium. Intracellular levels of potassium decrease and the cells depolarise. Digoxin increases this effect.
Both digoxin and hypomagnesemia inhibit the Na-K-pump resulting in decreased intracellular potassium.

Hypocalcemia (choice B) can result from magnesium deficiency. Release of calcium from the sarcoplasmic reticulum is inhibited by magnesium.

Low levels of magnesium stimulate the release of calcium and thereby an intracellular level of calcium. This effect similar to calcium inhibitors
makes it "nature's calcium inhibitor." Lack of magnesium inhibits the release of parathyroid hormone, which can result in hypoparathyroidism
and hypocalcemia. Furthermore, it makes skeletal and muscle receptors less sensitive to parathyroid hormone.

Hypokalemia (chocie C) can result from magnesium deficiency. Potassium channels are inhibited by magnesium. Hypomagnesemia results in
increased efflux of intracellular K. The cell loses potassium which then is excreted by the kidneys, resulting in hypokalemia.

Confusion (choice E) can result from magnesium deficiency. The neurological effects of magnesium deficiency are due to reduced electrical
excitation and blocked release of acetylcholine and N-methyl-D-aspartate (NMDA) glutamate receptors, an excitatory neurotransmitter of the
central nervous system.

A 43-year-old retired air hostess with chronic kidney disease on an aggressive hemodialysis program arrives at the emergency department
complaining of dark urine. The patient says that this is the first time that she has ever had this problem. On physical exam, there is noted
muscle weakness in all extremities. His urinalysis shows:

blood++
protein ++
glucose neg
leukocycte esterase neg
nitrites neg

Urine microscopy is negative for RBCs and shows some epithelial cells. Blood tests are notable for CK 1000 U/L and low serum phosphate.
Complications of severe hypophosphatemia include all of the following, except?
1)
2)
3)
4)
5)

Increased serum CPK enzyme


Respiratory muscle weakness
Intra-vascular hemolysis
Hypocalciuria
Cardiac dysrrhythmias

The correct answer is choice D

Hypophosphatemia is an electrolyte disturbance in which there is an abnormally low level of phosphate in the blood. The condition has many
causes, but is most commonly seen when malnourished patients (especially chronic alcoholics) are given large amounts of carbohydrates,
which creates a high phosphorus demand by cells, removing phosphate from the blood (refeeding syndrome). Because a decrease in
phosphate in the blood is sometimes associated with an increase in phosphate in the urine, the terms hypophosphatemia and "phosphaturia"
are occasionally used interchangeably.

Primary hypophosphatemia is the most common cause of nonnutritional rickets. Laboratory findings include low-normal serum calcium and
hypercalciuria, moderately low serum phosphate, elevated serum alkaline phosphatase, and low serum 1,25 dihydroxy-vitamin D levels,
hyperphosphaturia, and no evidence of hyperparathyroidism. Primary hypophosphatemia is also characterized by direct excess excretion of
phosphate by the kidneys, as from primary renal dysfunction, and also the direct action of many classes of diuretics on the kidneys. Additionally,
secondary causes, including both types of hyperparathyroidism cause hyperexcretion of phosphate in the urine.

The pathophysiology of hypophosphatemia is caused by the following three mechanisms:

Inadequate intake (often unmasked in refeeding after long-term low phosphate intake)

Increased excretion (e.g. in hyperparathyroidism)

Shift from extracellular to intracellular space (seen in treatment of diabetic ketoacidosis, refeeding, short-term increases in cellular
demand (e.g., hungry bones syndrome) and acute respiratory alkalosis)

Major symptoms of hypophosphatemia include muscle dysfunction and weakness. This occurs in major muscles, but also may manifest as
diplopia, low cardiac output, dysphagia, and respiratory depression due to respiratory muscle weakness. Mental status changes. This may
range from irritability to gross confusion, delirium, and coma. White cell dysfunction with subsequent worsening of infections. Instability of cell
membranes due to low ATP levels leading to rhabdomyolysis with increased CPK.

Increased serum CPK enzyme (choice A) is a common finding of hypophosphatemia due to the instability of cell membranes. Low ATP levels
lead to rhabdomyolysis with increased CPK.

Respiratory muscle weakness (chocice B) is a common physical finding of hypophosphatemia.

Intra-vascular hemolysis (choice C) is a common finding of hypophosphatemia due to the instability of cell membranes from low ATP levels.

Cardiac dysrrhythmias (choice E) is a common finding of hypophosphatemia due to dysfunction and weakness of cardiac tissue from low ATP
levels that results in low cardiac output and dysrrhythmias.

You are assisting with event coverage for a benefit marathon run (26.2 miles). The air temperature is 70F and the relative humidity is 60% at
the 9 AM start time. Five and a half hours later, a 28-year-old woman crosses the finish line. She is confused and unsteady on her feet. Her
friends bring her to the medical tent for evaluation, and report that she was drinking well at all water stations until the very end of the race.
Examination finds her to be disoriented to place, day, and situation. She appears well hydrated, with some edema noted on her hands and feet.
Her rectal temperature is 99.7F, and her reflexes are hypoactive. Her glucose level is 76 mg/dL. She then vomits.
Which of the following is the most likely laboratory finding in this patient?
1)
2)
3)
4)
5)

Sodium 115 mEq/L


Potassium 1.9 mEq/L
Calcium 12.9 mg/dL
Bicarbonate 7 mEq/L
Creatine phosphokinase (CPK) 10,250 U/L

The correct answer is choice A.

Patients - especially women - who participate in endurance athletic events are at risk for exercise-associated hyponatremia (or "marathon
hyponatremia"). This is caused by excessive hypotonic fluid intake as well as elevated circulating levels of ADH.

Hyponatremia can develop rapidly and can be quite severe, leading in many cases to cerebral edema and cardiac arrest. Once the diagnosis is
established, treatment should be provided with hypertonic (3%) sodium chloride.

Hypokalemia (choice B) with a potassium of 1.9 mEq/L is possible in an athlete who has excessive losses of potassium in her sweat. However,
hypokalemia would not explain this patient's other findings, which are consistent with volume overload and cerebral edema.

Hypercalcemia (choice C) would not be expected in this scenario, nor would it cause this patient's symptoms. A useful mnemonic for recalling
the symptoms of hypercalcemia is "Bones (bone pain), Stones (nephrolithiasis), Abdominal Moans (abdominal pain), and Psychiatric
Undertones (confusion and psychosis)."

A bicarbonate of 7 mEq/L (choice D) would indicate a severe metabolic acidosis. This would be possible in a patient with lactic acidosis and
shock - but this patient is noted as appearing well hydrated.

A creatine phosphokinase of 10,250 U/L (choice E) would indicate rhabdomyolysis, which is certainly a possibility in a patient who has just
completed a marathon. Patients with rhabdomyolysis may manifest symptoms of acute renal failure, but would not be expected to demonstrate
the constellation of findings in this patient, all of which point to marathon hyponatremia as the cause.

A 67 year old man comes to the office complaining of arthritis and related pains in his joints for the past several months. On further questioning,
he has been having severe headaches, and hearing loss. He feels somnolenscent throughout the day and the drowsiness does no go away with
changes in his sleep habits. He also comments that he has bought new hats and shoes the past several months because his old clothing items
havent been fitting him that well. Blood tests reveal everything within normal limits except very high alkaline phosphatase.
Normal serum calcium/normal phosphorus/very high alkaline phosphatase are associated with which of the following conditions?
1)
2)
3)
4)
5)

Sarcoidosis
Malignancy
Primary hypophosphatemia
Pagets disease
Milk-alkali syndrome

The correct answer is choice D

Pagets disease is a characterized by high bone-turnover rate that leads to hypercalecmia. The pathogenesis of Paget's disease is described in
3 stages which are:osteoclastic activity, mixed osteoclastic-osteoblastic activity, and finally exhaustive (burnt out) stage. Initially, there is a
marked increase in the rate of bone resorption at localized areas caused by large and numerous osteoclasts. These localized areas of
osteolysis are seen radiologically as an advancing lytic wedge in long bones or osteoporosis circumscripta in the skull. The osteolysis is
followed by a compensatory increase in bone formation induced by osteoblasts recruited to the area. This is associated with accelerated
deposition of lamellar bone in a disorganized fashion. This intense cellular activity produces a chaotic picture of trabecular bone ("mosaic"
pattern), rather than the normal linear lamellar pattern. The resorbed bone is replaced and the marrow spaces are filled by an excess of fibrous
connective tissue with a marked increase in blood vessels, causing the bone to become hypervascular. The bone hypercellularity may then
diminish leaving a dense "pagetic bone," also known as burned-out Paget disease.

An elevated level of alkaline phosphatase in the blood in combination with normal calcium, phosphate, and aminotransferase levels in an elderly
patient are suggestive of Paget's disease. Many patients do not know they have Paget's disease because they have a mild case with no
symptoms. Sometimes, symptoms may be confused with those of arthritis or other disorders. In other cases, the diagnosis is made only after
complications have developed. Symptoms can include:

Bone pain is the most common symptom. Bone pain can occur in any bone affected by Paget's disease. It often localizes to areas
adjacent to the joints.

Headaches and hearing loss may occur when Paget's disease affects the skull.

Pressure on nerves may occur when Paget's disease affects the skull or spine.

Somnolence (drowsiness) due to vascular steal syndrome of the skull.

Paralysis due to vascular steal syndrome of the vertebrae.

Increased head size, bowing of limb, or curvature of spine may occur in advanced cases.

Hip pain may occur when Paget's disease affects the pelvis or thighbone.

Damage to joint cartilage may lead to arthritis.

Teeth may spread intraorally due to the intraoral force placed on the anterior teeth (especially maxillary central and lateral incisors) by the
labial tissues, especially the muscles.

Chalkstick fractures.

Mosaic bone pattern

Hypercementosis in teeth may occur

sarcoidosis (choice A) frequently causes a dysregulation of vitamin D production with an increase in extrarenal production. Specifically,
macrophages inside the granulomas convert vitamin D to its active form, resulting in elevated levels of the hormone 1,25-dihydroxyvitamin D
and symptoms of hypervitaminosis D that may include fatigue, lack of strength or energy, irritability, metallic taste, temporary memory loss or
cognitive problems.

Malignancy (choice B) is a common cause of hypercalcemia through the production of PTH related petide secreting tumors (PTHrP). PTHrP
secreting tumors are often solid tumour with metastasis (e.g. breast cancer or classically squamous cell carcinoma), solid tumour with humoral
mediation of hypercalcaemia (e.g. lung or kidney cancer, pheochromocytoma), and hematological malignancies (multiple myeloma, lymphoma,
leukemia).

Primary hypophosphatemia (choice C) is the most common cause of nonnutritional rickets. Laboratory findings include low-normal serum
calcium and hypercalciuria, moderately low serum phosphate, elevated serum alkaline phosphatase, and low serum 1,25 dihydroxy-vitamin D
levels, hyperphosphaturia, and no evidence of hyperparathyroidism. Primary hypophosphatemia is also characterized by direct excess excretion
of phosphate by the kidneys, as from primary renal dysfunction, and also the direct action of many classes of diuretics on the kidneys.
Additionally, secondary causes, including both types of hyperparathyroidism cause hyperexcretion of phosphate in the urine.

Milk-alkali syndrome (chocie E) is characterized by hypercalcemia caused by repeated ingestion of calcium and absorbable alkali (such as
calcium carbonate, or milk and sodium bicarbonate). The name "milk-alkali syndrome" derives from when patients would take in excessive
amounts of milk and antacids to control their dyspepsia. Phosphorus and alkaline phosphatase levels are normal.

A 35-year-old woman presents to her physician complaining of headaches, increased sweating, and intermittent stabbing pain on her right side.
She has no chronic medical conditions, does not take any medication, and does not smoke or use other drugs or alcohol. Her family history is
significant for thyroid cancer on her maternal side, type unknown. On physical examination, the patient is pale and anxious appearing. Her blood
pressure is 210/130 and her heart rate is 120.
Which of the following is the most likely cause of hypercalcemia in the outpatient setting?
1)
2)
3)
4)
5)

Acromegaly
Malignancy
Granulomatous Disease
Pagets disease
Primary hyperparathyroidism

The correct answer is choice E

This patient most likely has multiple endocrine neoplasia (MEN) type II, also known as Sipples syndrome. It is characterized by medullary
carcinomaof the thyroid, pheochromocytoma, and parathyroid tumor or adenoma. In this case, the clinical history includes several

hyperadrenergic symptoms consistent with pheochromocytoma, including marked hypertension, tachycardia, pallor, and diaphoresis. Additional
symptoms of pheochromocytoma include palpitations, anxiety, and weight loss. A common manifestation of hyperparathyroidism is
hypercalcemia, which can result in kidney stones and flank pain. These findings, when coupled with a family history of thyroid cancer, should
make one suspicious of MEN type II. Primary hyperparathyroidism is the most common cause of hypercalcemia in the outpatient setting.

Hypercalcaemia is an elevated calcium level in the blood ((Normal range: 9-10.5 mg/dL). It can be an asymptomatic laboratory finding.
However, an elevated calcium level is often indicative of other diseases and a differential diagnosis should be undertaken if it persists. It can be
due to excessive skeletal calcium release, increased intestinal calcium absorption, or decreased renal calcium excretion.

EKG findings of a shortened QT interval and a widened T wave suggest hypercalcaemia and can lead to abnormal heart rhythms. Finally, peptic
ulcers may also occur. Symptoms are more common at high calcium blood values (12.0 mg/dLl). Severe hypercalcaemia (above 1516 mg/dL)
is considered a medical emergency: at these levels, coma and cardiac arrest can result. Initial therapy consists of IV fluids and diuretics
(furosemide) supplemented with bisphosphonates and calcitonin.

The general mnemonic for remembering the physical symptoms of hypercalcaemia: "groans (constipation), moans (psychotic noise), bones
(bone pain, especially if PTH is elevated), stones (kidney stones), and psychiatric overtones (including depression and confusion)." Other
symptoms can include fatigue, anorexia, nausea, vomiting, pancreatitis and increased urination.

Acromegaly (choice A) is a syndrome that results when the pituitary gland produces excess growth hormone (hGH) after epiphyseal plate
closure at puberty. Although hypercalciuria is common in acromegaly, hypercalcemia is rare; its presence should suggest that the patient has
multiple endocrine neoplasia (MEN I syndrome). A number of disorders may increase the pituitary's GH output, although most commonly it
involves a GH producing tumor called pituitary adenoma, derived from a distinct type of cell (somatotrophs). Acromegaly most commonly affects
adults in middle age, and can result in severe disfigurement, serious complicating conditions, and premature death if unchecked. Because of its
insidious pathogenesis and slow progression, the disease is hard to diagnose in the early stages and is frequently missed for many years, until
changes in external features, especially of the face, become noticeable.

Malignancy (choice B) is the most common cause of hypercalcemia in the elderly through the production of PTH related petide secreting tumors
(PTHrP). PTHrP secreting tumors are often solid tumour with metastasis (e.g. breast cancer or classically squamous cell carcinoma), solid
tumour with humoral mediation of hypercalcaemia (e.g. lung or kidney cancer, pheochromocytoma), and hematological malignancies (multiple
myeloma, lymphoma, leukemia).

Granulomatous Disease (choice C) such as as sarcoidosis can result in hypercalcemia. Granulomas have elevated levels of 1-alpha
hydroxylase, an enzyme normally found in the renal proximal tubules that is responsible for the final step in biosynthesis of Vitamin D3. Elevated
levels of Vitamin D cause hypercalcemia by increasing calciums absorption in the gut and decreasing its excretion by the kidneys.

Pagets disease (choice D) is a characterized by high bone-turnover rate that leads to hypercalecmia. The pathogenesis of Paget's disease is
described in 3 stages which are:osteoclastic activity, mixed osteoclastic-osteoblastic activity, and finally exhaustive (burnt out) stage. Initially,
there is a marked increase in the rate of bone resorption at localized areas caused by large and numerous osteoclasts. These localized areas of
osteolysis are seen radiologically as an advancing lytic wedge in long bones or osteoporosis circumscripta in the skull. The osteolysis is
followed by a compensatory increase in bone formation induced by osteoblasts recruited to the area. This is associated with accelerated
deposition of lamellar bone in a disorganized fashion. This intense cellular activity produces a chaotic picture of trabecular bone ("mosaic"
pattern), rather than the normal linear lamellar pattern. The resorbed bone is replaced and the marrow spaces are filled by an excess of fibrous
connective tissue with a marked increase in blood vessels, causing the bone to become hypervascular. The bone hypercellularity may then
diminish leaving a dense "pagetic bone," also known as burned-out Paget disease.

A 55-year-old man presents to the clinic complaining of weakness. He is a known hypertensive currently being managed on lisinopril.
Examination findings were normal except for an irregular pulse. The results of his laboratory investigations are as follows:
Hematocrit - 40%
Random blood sugar - 85mg/dL
Total serum proteins - 6.8g/dL
Albumin - 3.7 g/dL
Globulin - 3.1 g/dL Sodium - 140 mEq/L
Chloride - 102 mEq/L
Potassium - 6.2 mEq/L
Bicarbonate - 16 mEq/L
Blood pH - 7.30 BUN 13mg/dL
Serum uric acid 4mg/dL
Chest x-ray findings were normal and EKG revealed tall peaked T-waves.
Which of the following is the next best response?
1) Prepare for dialysis
2) Restrict potassium intake
3) Give IV calcium gluconate

4) Repeat electrolytes
5) Continue observing the patient
The correct answer is Choice C.

The patient above is receiving a potassium sparing diuretic and has developed hyperkalemia, cardiotoxicity and metabolic acidosis. However,
he is not in renal failure. IV calcium gluconate should be given to the patient immediately to protect the heart before commencing IV therapy for
reducing serum potassium.

Figure 1: Causes of Hyperkalemia

A 43-year-old woman presents with bone pain, joint pain, and fatigue. Further history reveals blood in her urine and difficulty moving her bowels.
She has lost over ten pounds unintentionally with poor appetite and night sweats. She has no chronic medical conditions and does not take any

medications or use drugs. Her family history is negative for malignancy. Physical examination is positive for lymphadenopathy or
hepatosplenomegaly.
Which of the following is the most likely cause of hypercalcemia in the elderly?
1)
2)
3)
4)
5)

Acromegaly
Malignancy
Granulomatous Disease
Pagets disease
Primary hyperparathyroidism

The correct answer is choice B

This patient most likely has malignancy such as a squamous cell carcinoma. Malignancy is the most common cause of hypercalcemia in the
elderly through the production of PTH related petide secreting tumors (PTHrP). PTHrP secreting tumors are often solid tumour with metastasis
(e.g. breast cancer or classically squamous cell carcinoma), solid tumour with humoral mediation of hypercalcaemia (e.g. lung or kidney cancer,
pheochromocytoma), and hematological malignancies (multiple myeloma, lymphoma, leukemia).

Hypercalcaemia is an elevated calcium level in the blood ((Normal range: 9-10.5 mg/dL). It can be an asymptomatic laboratory finding.
However, an elevated calcium level is often indicative of other diseases and a differential diagnosis should be undertaken if it persists. It can be
due to excessive skeletal calcium release, increased intestinal calcium absorption, or decreased renal calcium excretion.

EKG findings of a shortened QT interval and a widened T wave suggest hypercalcaemia and can lead to abnormal heart rhythms. Finally, peptic
ulcers may also occur. Symptoms are more common at high calcium blood values (12.0 mg/dLl). Severe hypercalcaemia (above 1516 mg/dL)
is considered a medical emergency: at these levels, coma and cardiac arrest can result. Initial therapy consists of IV fluids and diuretics
(furosemide) supplemented with bisphosphonates and calcitonin.

The general mnemonic for remembering the physical symptoms of hypercalcaemia: "groans (constipation), moans (psychotic noise), bones
(bone pain, especially if PTH is elevated), stones (kidney stones), and psychiatric overtones (including depression and confusion)." Other
symptoms can include fatigue, anorexia, nausea, vomiting, pancreatitis and increased urination.

Acromegaly (choice A) is a syndrome that results when the pituitary gland produces excess growth hormone (hGH) after epiphyseal plate
closure at puberty. Although hypercalciuria is common in acromegaly, hypercalcemia is rare; its presence should suggest that the patient has
multiple endocrine neoplasia (MEN I syndrome). A number of disorders may increase the pituitary's GH output, although most commonly it
involves a GH producing tumor called pituitary adenoma, derived from a distinct type of cell (somatotrophs). Acromegaly most commonly affects
adults in middle age, and can result in severe disfigurement, serious complicating conditions, and premature death if unchecked. Because of its
insidious pathogenesis and slow progression, the disease is hard to diagnose in the early stages and is frequently missed for many years, until
changes in external features, especially of the face, become noticeable.

Granulomatous Disease (choice C) such as as sarcoidosis can result in hypercalcemia. Granulomas have elevated levels of 1-alpha
hydroxylase, an enzyme normally found in the renal proximal tubules that is responsible for the final step in biosynthesis of Vitamin D3. Elevated
levels of Vitamin D cause hypercalcemia by increasing calciums absorption in the gut and decreasing its excretion by the kidneys.

Pagets disease (choice D) is a characterized by high bone-turnover rate that leads to hypercalecmia. The pathogenesis of Paget's disease is
described in 3 stages which are:osteoclastic activity, mixed osteoclastic-osteoblastic activity, and finally exhaustive (burnt out) stage. Initially,
there is a marked increase in the rate of bone resorption at localized areas caused by large and numerous osteoclasts. These localized areas of
osteolysis are seen radiologically as an advancing lytic wedge in long bones or osteoporosis circumscripta in the skull. The osteolysis is
followed by a compensatory increase in bone formation induced by osteoblasts recruited to the area. This is associated with accelerated
deposition of lamellar bone in a disorganized fashion. This intense cellular activity produces a chaotic picture of trabecular bone ("mosaic"
pattern), rather than the normal linear lamellar pattern. The resorbed bone is replaced and the marrow spaces are filled by an excess of fibrous
connective tissue with a marked increase in blood vessels, causing the bone to become hypervascular. The bone hypercellularity may then
diminish leaving a dense "pagetic bone," also known as burned-out Paget disease.

Primary hyperparathyroidism (choice E) causes hypercalcemia through the excessive secretion of parathyroid hormone (PTH), usually by an
adenoma (benign tumors) of the parathyroid glands. Primary hyperparathyroidism is also a feature of several familial endocrine disorders:
Multiple endocrine neoplasia type 1 and type 2A (MEN type 1 and MEN type 2A), and familial hyperparathyroidism. It is the most common
cause of hypercalcemia in the outpatient setting.

A 28-year-old pregnant woman at 36 weeks gestation presents to the ER with loss of appetite, drowsiness, confusion and cramps. Examination
revealed normal vital signs and no uterine contractions. Random blood glucose was 110mg/dL and serum electrolytes yielded the following:
Sodium - 105 mEq/L
Chloride - 96 mEq/L

Potassium - 4.2mEq/L
Bicarbonate - 23 mEq/L
Which of the following would be most appropriate in his management?
1)
2)
3)
4)
5)

Induce delivery
Administer IV normal saline
Restrict fluids
Prepare for emergency Caesarian section
Observe the patient

The correct answer is Choice C.

The patient above is hyponatremic, which may be attributable to oxytocin. With normal vital signs, we can assume that she is euvolemic. This
can be corrected with fluid restriction. IV normal saline is appropriate in the setting of hypovolemic hyponatremia, and in this setting, if
administered too quickly, could result in central pontine myelinolysis.

You are seeing a 6 month old female infant in your office for an initial checkup. The family recently emigrated from Western Africa, and the child
has not previously seen a physician or received any immunizations. With the assistance of a translator, you learn that the mother is concerned
that the infant appears "weak" for the past month, and that the infant's father died of a "brain infection" last year after a long illness. On
examination, you note a thin infant with sallow skin. Her length is 63.5 cm (15th percentile) with weight 5.7 kg (10 kg). The heart tones are
normal, and the lung fields are clear. The liver edge is palpable 3 cm below the right costal margin, and the abdomen is somewhat distended

but soft.
Which of the following is the most appropriate next step in the management of this patient?
1)
2)
3)
4)
5)

Place tuberculin purified protein derivative (PPD) skin test


Obtain enzyme-linked immunosorbent assay (ELISA) for HIV
Obtain HIV RNA polymerase chain reaction (PCR)
Check adenosine deaminase (ADA) level
Perform lumbar puncture and initiate empiric antibiotics

The correct answer is choice C.

Everything in this vignette suggests the possibility of HIV infection in this infant, from their recent emigration from Africa, to the father's untimely
death, to the appearance of primary failure to thrive. What this question is really asking, then, is what is the best test in a patient of this age to
confirm a diagnosis of HIV?

Remember that IgG is transmitted across the placenta during the third trimester. If this patient's mother is also infected with HIV and has
mounted an antibody response, then it would be expected that the infant would have some level of circulating antibody in her blood, as well.
Thus, while antibody-based tests for HIV (such as ELISA, EIA, and Western blot) are appropriate screening tools for most patients, they should
not be used in young infants. False positives can occur up to 18 months of age due to persistent maternal antibody; therefore, the best test one
that directly measures the HIV viral genome such as an HIV DNA PCR (qualitative) or HIV RNA PCR (quantitative).

Placing a PPD (choice A) is not the best choice at this point. Patients with HIV are certainly at risk for tuberculosis, but you should confirm the
diagnosis of HIV first. Similarly, while TB could cause failure to thrive, it does not seem as likely as HIV in this scenario.

The HIV ELISA (choice B) is a good screening test for most patients in whom a diagnosis of HIV is being considered. However, for the reasons
described above, a positive result in this case could be due to maternal antibody or true infection - a different test is needed.

A low adenosine deaminase level (choice D) is seen in many patients with severe combined immunodeficiency (SCID), and these patients will
get many of the same opportunistic infections seen in patients with HIV. However, given the suggestive history, you should first evaluate the

possibility of HIV infection.

Performing a lumbar puncture and starting antibiotics (choice E) is not necessary. This infant has been ill for some time but is still alive, which is
decidedly not the natural history of untreated bacterial meningitis.

A 40-year-old man presents to your clinic with weakness and constipation. He is a chronic alcoholic, and has been experiencing nausea and
vomiting for the past 2 weeks. His vital signs revealed a temperature of 36.6C, pulse of 120/min, blood pressure of 80/50mmHg and respiration
of 12 cycles/min. His abdomen was non-tender and silent. Random blood glucose was 85mg/dL. His EKG showed a loss of T-wave.
Which of the following would be most appropriate in his management?
1)
2)
3)
4)
5)

Restrict fluids
Administer oral potassium supplements
Administer IV potassium in glucose solution containing insulin
Administer IV frusemide
Give the patient an enema

The correct answer is choice B.

Vomiting leads to hypokalemia through oral and urinary loses of potassium. This may result in muscle paralysis, in this case manifesting as
ileus, hypotension and characteristic EKG changes. Serum electrolytes should be investigated and if hypokalemia is present, it should be
corrected with potassium supplements. Oral administration is preferred over IV because of potentially fatal arrhythmias that could arise.

Causes of hypokalemia:

Intake (never alone)

Shift

Treatment with insulin

Alkalosis

-2 stimulation

Periodic paralysis

Treatment of anemia

Increased Excretion

GI

Renal (Hyperaldosteronism, Diuresis, Ampho-B, Hypomagnesemia)

Clinical Effects:

Cardiac arrhythmias

Muscle weakness

Rhabdomyolysis

Renal dysfunction

Glucose intolerance

You were called to see a 38-year-old female inpatient that was admitted for cancer chemotherapy on account of AML. During the encounter,
you discover that she had no complaints, normal vital signs and physical findings. However, the results of her most recent laboratory
investigations are as follows:

Hematocrit - 38%
RBC - 5 million/mm3
WBC - 6,000/mm3
Neut - 60%
Lymph - 33%
Mono - 5%
Eosin - 2% Random blood sugar - 110mg/dL
Sodium - 138 mEq/L
Chloride - 99 mEq/L
Potassium - 6.5 mEq/L
Bicarbonate - 23 mEq/L
Blood pH - 7.38 Total serum proteins - 6.5g/dL
Albumin - 3.8 g/dL

Globulin - 2. 7 g/dL
EKG and chest x-ray findings were normal

Which of the following is the next best response?


1)
2)
3)
4)
5)

Prepare for dialysis


Restrict potassium intake
Give IV calcium gluconate
Repeat serum electrolytes
Continue observing the patient

The correct answer is choice D

The patient above is asymptomatic, and seems to have elevated potassium. With hyperkalemia, one would expect to see an acidic pH, a low
bicarbonate level and probably EKG changes (tall peaked T waves, prolonged PR interval, etc). However, when serum potassium appears to be
high in an asymptomatic patient, one should suspect that the RBCs in the sample hemolysed and thus resulted in a falsely elevated potassium.
A repeat investigation should be requested.

Figure 1: Causes of Hyperkalemia

A 25-year-old male patient complains of increased thirst and increased frequency of urination. He denies any trauma or recent surgery. A urine
sample is taken and urine specific gravity is 1.001 and osmolality is 350. He is admitted to the hospital for observation and water restriction.

After 6 hours, his urine specific gravity is 1.003 and his serum osmolality is 365. He is given intranasal desmopressin (DDAVP) and his urine
specific gravity remains unchanged while his serum osmolality increases to 400. The patient becomes increasingly lethargic and irritable and
eventually has a tonic clonic seizure. A stat metabolic panel is obtained and his plasma sodium comes back at 158 mEq/L.
What medication most likely caused his hypernatremia?
1)
2)
3)
4)
5)

Aspirin
Furosemide
Gentamicin
Lithium
Hydrochlorothiazide

The correct answer is choice D

This patient is suffering from nephrogenic diabetes insipidus which is a side effect of lithium. Diabetes insipidus is characterized by increased
thirst (polydipsia) and increased urination (polyuria) with the inability to to concentrate urine due to a lack of ADH. Therefore, urine specific
gravity will be low (<1.006) and serum osmolality will be >250. Nephrogenic diabetes insipidus (lack of response to ADH) can be distinguished
from central (loss of ADH production) because nephrogenic DI will not respond to DDAVP which is a ADH analog.

Clinical manifestations of hypernatremia can be subtle, consisting of lethargy, weakness, irritability, and edema. With more severe elevations of
the sodium level, seizures and coma may occur. Severe symptoms are usually due to acute elevation of the plasma sodium concentration to
above 158 mEq/L (normal is typically about 135-145 mEq/L). The differential diagnosis of hypernatremia can be narrowed by assessing the
patients volume status.

Hypovolemic

Inadequate intake of water, typically in elderly or otherwise disabled patients who are unable to take in water as their thirst dictates. This
is the most common cause of hypernatremia.

Excessive losses of water from the urinary tract, which may be caused by glycosuria, or other osmotic diuretics.

Water losses associated with extreme sweating.

Severe watery diarrhea

Euvolemic

Excessive excretion of water from the kidneys caused by diabetes insipidus, which involves either inadequate production of the
hormone,vasopressin, from the pituitary gland or impaired responsiveness of the kidneys to vasopressin.

Hypervolemic

Intake of a hypertonic fluid (a fluid with a higher concentration of solutes than the remainder of the body). This is relatively uncommon,
though it can occur after a vigorous resuscitation where a patient receives a large volume of a concentrated sodium bicarbonate solution.
Ingesting seawater also causes hypernatremia because seawater is hypertonic.

Mineralcorticoid excess due to a disease state such as Conn's syndrome or Cushing's Disease

Aspirin (choice A) can cause acute tubular necrosis (ATN) and acute interstitial nephritis thereby increasing serum creatinine but it would not

cause an low urine specific gravity or a high serum osmolality. It would also not cause hypernatremia.

Furosemide (choice B) is a loop diuretic which would cause decreased urine specific gravity but the serum osmolality would not increase as
much. It would also not cause hypernatremia.

Gentamicin (choice C) an cause acute tubular necrosis (ATN) thereby increasing serum creatinine but it would not cause an low urine specific
gravity or a high serum osmolality. It would also not cause hypernatremia.

Hydrochlorothiazide (choice E) would cause hyponatremia by making the body excrete more sodium in urine.

A 57 year old homeless man is rushed to the emergency department. The patient was found to be unresponsive. He has a history of multiple
alcohol related admissions, uncontrolled diabetes, dialysis dependent chronic renal failure. On physical exam, the patient is unarousable with
temperature of 37.2 degrees, BP 115/70, HR 95 and RR of 22. He complains of blurry vision but the rest of the exam is unremarkable.
A stat metabolic panel reveals:

Na: 137
K: 3.6
Cl: 95
HCO3: 17
BUN: 20
Cr: 6.7
Glucose: 235

In a patient with a wide anion gap metabolic acidosis who has a large osmolar gap, which of the following possible etiologies should be
seriously considered?
1)
2)
3)
4)

Diarrhea
salicylate overdose
methanol intoxication
uremia

5) diabetic ketoacidosis

The correct answer is choice C

To distinguish between the main types of metabolic acidosis, a clinical tool called the anion gap is considered very useful. It is calculated by
subtracting the chloride and bicarbonate levels from the sodium.

Anion gap = ( [Na+] ) - ( [Cl-]+[HCO3-] )

As sodium is the main extracellular cation, and chloride and bicarbonate are the main anions, the result should reflect the remaining anions.
Normally, this concentration is about 8-16 mmol/l (124). An elevated anion gap (i.e. > 16 mmol/l) can indicate particular types of metabolic
acidosis, particularly certain poisons, lactate acidosis and ketoacidosis.
Caluclating this patients anion gap reveals a normal anion gap, which shortens the differential diagnosis considerably.

Methanol has a high toxicity in humans. If ingested, as little as 10 mL can cause permanent blindness by destruction of the optic nerve and 30
mL is potentially fatal, although a fatal dose is typically 100125 mL. Toxic effects take hours to start and effective antidotes can often prevent
permanent damage. Because of its similarities to ethanol, it is difficult to differentiate between the two (such is the case with denatured alcohol).

Methanol is toxic by two mechanisms. Firstly, methanol (whether it enters the body by ingestion, inhalation, or absorption through the skin) can
be fatal due to its CNS depressant properties in the same manner as ethanol poisoning. Secondly, in a process of toxication, it is metabolised to
formic acid (which is present as the formate ion) via formaldehyde in a process initiated by the enzyme alcohol dehydrogenase in the liver. The
reaction to formate proceeds completely, with no detectable formaldehyde remaining. Formate is toxic because it inhibits mitochondrial
cytochrome c oxidase, causing the symptoms of hypoxia at the cellular level, and also causing metabolic acidosis among a variety of other
metabolic disturbances.

Methanol poisoning can be treated with the antidotes ethanol or fomepizole. Both of these drugs act to reduce the action of alcohol
dehydrogenase on methanol by means of competitive inhibition, so that it is excreted by the kidneys rather than being transformed into toxic
metabolites. Further treatment may include giving sodium bicarbonate for metabolic acidosis or hemodialysis to remove methanol and formate

from the blood.

The initial symptoms of methanol intoxication include central nervous system depression, headache, dizziness, nausea, lack of coordination,
confusion, and with sufficiently large doses, unconsciousness and death. The initial symptoms of methanol exposure are usually less severe
than the symptoms resulting from the ingestion of a similar quantity of ethanol. Once the initial symptoms have passed, a second set of
symptoms arises, 10 to as many as 30 hours after the initial exposure to methanol, including blurring or complete loss of vision and acidosis.
These symptoms result from the accumulation of toxic levels of formate in the bloodstream, and may progress to death by respiratory failure.

diarrhea (choice A) leads to a nongap acidosis from a loss of GI secretions which are rich in bicarbonate.

salicylate overdose (choice B) stimulate the respiratory center to cause hyperventilation (respiratory alkalosis) which is dose-dependent.
Metabolic acidosis is the most serious acid-base disorder and is due to increased production of endogenous acids rather than the salicylate
itself. The combination of metabolic acidosis and respiratory alkalosis can be a difficult situation to diagnose from the blood gases.

uremia (choice D) leads to a high gap anion acidosis and is a term used to loosely describe the illness accompanying kidney failure due to the
buildup of nitrogenous waste products. IIn kidney failure, urea and other waste products, which are normally excreted into the urine, are retained
in the blood. Early symptoms include anorexia and lethargy, and late symptoms can include decreased mental acuity and coma. It is usually
diagnosed in kidney dialysis patients when the glomerular filtration rate, a measure of kidney function, is below 50% of normal. Uremia can also
result in fibrinous pericarditis. There are many dysfunctions caused by uremia affecting many systems of the body, such as blood (lower levels
of erythropoietin), sex (lower levels of testosterone/estrogen) and bones (osteoporosis and metastatic calcifications).

diabetic ketoacidosis (chocie E) is a type of metabolic acidosis associated with high concentrations of ketone bodies, formed by the breakdown
of fatty acids and the deamination of amino acids. The two common ketones produced in humans are acetoacetic acid and -hydroxybutyrate.
In diabetic patients, ketoacidosis is usually accompanied by insulin deficiency, hyperglycemia, and dehydration. Particularly in type 1 diabetics
the lack of insulin in the bloodstream prevents glucose absorption and can cause unchecked ketone body production (through fatty acid
metabolism) potentially leading to dangerous glucose and ketone levels in the blood. Hyperglycemia results in glucose overloading the kidneys
and spilling into the urine (transport maximum for glucose is exceeded). Dehydration results following the osmotic movement of water into urine
(Osmotic diuresis), exacerbating the acidosis.

A 41 year old woman undergoes a subtotal thyroidectomy for removal of a malignant growth. On postop day one, she complains of muscle

cramps and is noted to have a twitch in her cheek when the facial nerve is tapped. Further physical examination reveals petechia. When the
blood pressure cuff is inflated to take her blood pressure, a carpal spasm is noted. A stat EKG is ordered which shows a prolonged QT interval.
Symptoms of hypocalcemia in patients with history of thyroidectomy are most likely due to?
1)
2)
3)
4)
5)

Osteomalacia
hypovitaminosis D
DiGeorge syndrome
Hypoparathyroidism
rapid blood transfusion during thyroidectomy

The correct answer is choice D

A thyroidectomy is an operation that involves the surgical removal of all or part of the thyroid gland. Surgeons often perform a thyroidectomy
when a patient has thyroid cancer or some other condition of the thyroid gland (such as hyperthyroidism). Other indications for surgery include
cosmetic (very enlarged thyroid), or symptomatic obstruction (causing difficulties in swallowing or breathing). One of the complications of
"thyroidectomy" is transient hypoparathyroidism that can lead to hypocalcemia.

Hypocalcaemia is the presence of low serum calcium levels in the blood, usually taken as less than 2.1 mmol/L or 9 mg/dl or an ionized calcium
level mm of less than 1.1 mmol/L (4.5 mg/dL). About half of all calcium is bound to proteins such as serum albumin, but it is the unbound, or
ionized, calcium that the body regulates. If a person has abnormal levels of blood proteins, then the plasma calcium may be inaccurate. In the
setting of low serum albumin (frequently seen in patients with chronic diseases, hepatic disease or even long term hospitalization), the formula
for corrected calcium is: CorrCa = Measured serum Ca + [(4.0 - measured serum albumin) * 0.8]. Thus, if the albumin is low, the measured
calcium may appear low when in fact it is physiologically within normal limits.

Physical signs of hypocalcemia can include petechia, perioral tingling and paraesthesia and tetany.

Trousseau sign of latent tetany (eliciting carpal spasm by inflating the blood pressure cuff and maintaining the cuff pressure above systolic),
tendon reflexes are hyperactive, and Chvostek's sign (tapping of the inferior portion of the zygoma will produce facial spasms). On EKG, the QT
interval is often prolonged.

To correct hypocalcemia, intravenous calcium gluconate is given slowly in a period of 10 minutes, or if the hypocalcaemia is severe, calcium
chloride is given instead. Maintenance doses of both calcium and vitamin-D (often as 1,25-(OH)2-D3, i.e. calcitriol) are often necessary to
prevent further decline.

Osteomalacia (choice A) is the softening of the bones due to defective bone mineralization. Osteomalacia in children is known as rickets, and
because of this, use of the term osteomalacia is often restricted to the milder, adult form of the disease. It may show signs as diffuse body
pains, muscle weakness, and fragility of the bones. A common cause of the disease is a deficiency in vitamin D, which is normally obtained from
the diet and/or sunlight exposure. Osteomalacia is not a complication of thyroidectomy.

Hypovitaminosis D (choice B) is a deficiency of Vitamin D. It can result from: inadequate nutritional intake of vitamin D coupled with inadequate
sunlight exposure (in particular sunlight with adequate ultra violet B rays), disorders that limit vitamin D absorption, and conditions that impair
the conversion of vitamin D into active metabolites including certain liver, kidney, and hereditary disorders. Deficiency results in impaired bone
mineralization and leads to bone softening diseases including rickets in children and osteomalacia and osteoporosis in adults. Hypovitaminosis
D is not a complication of thyroidectomy.

DiGeorge syndrome (choice C) is caused by genetic deletions found on the long arm of one of the two 22nd chromosomes. 22q11.2 deletion
syndrome may involve migration defects of neural crest-derived tissues, particularly affecting development of the third and fourth branchial
pouches (pharyngeal pouches). This affects the thymus gland; a mediastinal organ largely responsible for differentiation and induction of
tolerance in T-cells, and the parathyroid glands, responsible for regulation of blood calcium levels. The hypocalcaemia that results requires
lifelong vitamin D and calcium supplements.

Rapid blood transfusion during thyroidectomy (choice E) are generally very rare causes of transient hypocalcemia that rarely necessitates
treatment. Potassium levels are more affected by transfusions. Patients with renal failure may have elevated K if transfused with blood stored
for > 1 wk (K accumulation is usually insignificant in blood stored for < 1 wk). Mechanical hemolysis during transfusion may increase K.
Hypokalemia may occur about 24 h after transfusion of older RBCs (> 3 wk), which take up K.

A 67 year old African-American woman presents to her primary care doctor complaining of vision problems, fatigue, and constipation for the
past six months. Her physical exam is unremarkable except for tender red skin nodules. Serum chemistry is remarkable for elevated calcium at
11.6, low phosphate at 2.1, slightly elevated alkaline phosphatase at 98, and elevated 1-alpha hydroxylase.

Which of the following is the most likely cause of the patients hypercalcemia?
1)
2)
3)
4)
5)

Primary/Secondary hyperparathyroidism
Malignancy
Granulomatous Diseas
Pagets disease
Milk-alkali syndrome

The correct answer is choice C

This patient most likely has sarcoidosis. African Americans are at higher risk for this disease compared to the general population. Her symptoms
are consistent with hypercalcemia, which may cause CNS symptoms. The skin rash described in the case is erythema nodosum, and the vision
problems are most likely related to granulomas in the optic nerve. These physical findings should raise a suspicion of sarcoidosis. The
granulomas have elevated levels of 1-alpha hydroxylase, an enzyme normally found in the renal proximal tubules that is responsible for the final
step in biosynthesis of Vitamin D3. Elevated levels of Vitamin D cause hypercalcemia by increasing calciums absorption in the gut and
decreasing its excretion by the kidneys.

Hypercalcaemia is an elevated calcium level in the blood ((Normal range: 9-10.5 mg/dL). It can be an asymptomatic laboratory finding.
However, an elevated calcium level is often indicative of other diseases and a differential diagnosis should be undertaken if it persists. It can be
due to excessive skeletal calcium release, increased intestinal calcium absorption, or decreased renal calcium excretion.

EKG findings of a shortened QT interval and a widened T wave suggest hypercalcaemia and can lead to abnormal heart rhythms. Finally, peptic
ulcers may also occur. Symptoms are more common at high calcium blood values (12.0 mg/dLl). Severe hypercalcaemia (above 1516 mg/dL)
is considered a medical emergency: at these levels, coma and cardiac arrest can result. Initial therapy consists of IV fluids and diuretics
(furosemide) supplemented with bisphosphonates and calcitonin.

The general mnemonic for remembering the physical symptoms of hypercalcaemia: "groans (constipation), moans (psychotic noise), bones
(bone pain, especially if PTH is elevated), stones (kidney stones), and psychiatric overtones (including depression and confusion)." Other
symptoms can include fatigue, anorexia, nausea, vomiting, pancreatitis and increased urination.

Primary hyperparathyroidism (choice A) causes hypercalcemia through the excessive secretion of parathyroid hormone (PTH), usually by an

adenoma (benign tumors) of the parathyroid glands. Primary hyperparathyroidism is also a feature of several familial endocrine disorders:
Multiple endocrine neoplasia type 1 and type 2A (MEN type 1 and MEN type 2A), and familial hyperparathyroidism.

Malignancy (choice B) is a common cause of hyopercalcemia through the production of PTH related petide secreting tumors (PTHrP). PTHrP
secreting tumors are often solid tumour with metastasis (e.g. breast cancer or classically squamous cell carcinoma), solid tumour with humoral
mediation of hypercalcaemia (e.g. lung or kidney cancer, pheochromocytoma), and hematological malignancies (multiple myeloma, lymphoma,
leukemia).

Pagets disease (choice D) is a characterized by high bone-turnover rate that leads to hypercalecmia. The pathogenesis of Paget's disease is
described in 3 stages which are:osteoclastic activity, mixed osteoclastic-osteoblastic activity, and finally exhaustive (burnt out) stage. Initially,
there is a marked increase in the rate of bone resorption at localized areas caused by large and numerous osteoclasts. These localized areas of
osteolysis are seen radiologically as an advancing lytic wedge in long bones or osteoporosis circumscripta in the skull. The osteolysis is
followed by a compensatory increase in bone formation induced by osteoblasts recruited to the area. This is associated with accelerated
deposition of lamellar bone in a disorganized fashion. This intense cellular activity produces a chaotic picture of trabecular bone ("mosaic"
pattern), rather than the normal linear lamellar pattern. The resorbed bone is replaced and the marrow spaces are filled by an excess of fibrous
connective tissue with a marked increase in blood vessels, causing the bone to become hypervascular. The bone hypercellularity may then
diminish leaving a dense "pagetic bone," also known as burned-out Paget disease.

Milk-alkali syndrome (chocie E) is characterized by hypercalcemia caused by repeated ingestion of calcium and absorbable alkali (such as
calcium carbonate, or milk and sodium bicarbonate). The name "milk-alkali syndrome" derives from when patients would take in excessive
amounts of milk and antacids to control their dyspepsia.

A 37 year old African-American woman presents to the ED with abdominal pain for the past 48 hours. She has a history of sarcoidosis,
hypertyhroidism, squamous cell carcinoma, and hypertension controlled with thiazide diuretics. After several hours, her mental status
deteriorates and see becomes obtunded. On EKG, she has a shortened QT interval and widened T waves.
All of the following is contributing to the patients hypercalcemia except?
1)
2)
3)
4)
5)

Sarcoidosis
Malignancy
secondary hyperparathyroidism
thiazide use
hyperthyroidism

The correct answer is choice C

Hypercalcemia is an elevated calcium level in the blood (Normal range: 9-10.5 mg/dL or 2.2-2.6 mmol/L). It can be an asymptomatic laboratory
finding, but because an elevated calcium level is often indicative of other diseases, a diagnosis should be undertaken if it persists. It can be due
to excessive skeletal calcium release, increased intestinal calcium absorption, or decreased renal calcium excretion.

There is a general mnemonic for remembering the effects of hypercalcaemia: "groans (constipation), moans (psychotic noise), bones (bone
pain, especially if PTH is elevated), stones (kidney stones), and psychiatric overtones (including depression and confusion)." Other symptoms
can include fatigue, anorexia, nausea, vomiting, pancreatitis and increased urination.

Abnormal heart rhythms can result, and ECG findings of a short QT interval and a widened T wave suggest hypercalcaemia. Finally, peptic
ulcers may also occur. Symptoms are more common at high calcium blood values (12.0 mg/dL or 3 mmol/l). Severe hypercalcaemia (above 15
16 mg/dL or 3.75-4 mmol/l) is considered a medical emergency: at these levels, coma and cardiac arrest can result.

Although primary hyperparathyroidism account for the majority of cases of hypercalcemia, secondary hyperparathyroidism results in the
excessive secretion of parathyroid hormone (PTH) by the parathyroid glands in response to hypocalcemia and associated hypertrophy of the
glands. Chronic renal failure is the most common cause of secondary hyperparathyroidism. Failing kidneys do not convert enough vitamin D to
its active form, and they do not adequately excrete phosphorus. When this happens, insoluble calcium phosphate forms in the body and
removes calcium from the circulation. Both processes lead to hypocalcemia and hence secondary hyperparathyroidism.

sarcoidosis (choice A) frequently causes a dysregulation of vitamin D production with an increase in extrarenal production. Specifically,
macrophages inside the granulomas convert vitamin D to its active form, resulting in elevated levels of the hormone 1,25-dihydroxyvitamin D
and symptoms of hypervitaminosis D that may include fatigue, lack of strength or energy, irritability, metallic taste, temporary memory loss or
cognitive problems. Subsequently, patients develop hypercalcemia.

Malignancy (choice B) is a common cause of hypercalcemia through the production of PTH related petide secreting tumors (PTHrP). PTHrP
secreting tumors are often solid tumour with metastasis (e.g. breast cancer or classically squamous cell carcinoma), solid tumour with humoral
mediation of hypercalcaemia (e.g. lung or kidney cancer, pheochromocytoma), and hematological malignancies (multiple myeloma, lymphoma,
leukemia).

thiazide use (choice D) lower urinary calcium excretion, making them useful in preventing calcium-containing kidney stones. This effect is
associated with positive calcium balance and is associated with an increase in bone mineral density and reductions in fracture rates attributable
to osteoporosis. By a lesser understood mechanism, thiazides directly stimulate osteoblast differentiation and bone mineral formation, further
slowing the course of osteoporosis.

hyperthyroidism (choice E) leads to high bone turnover rates and subsequent hypercalcemia.

A 65-year-old man with mild heart failure is treated with a loop diuretic . A few days later the man complains of muscle weakness. Laboratory
results are shown below. Arterial PCO2: 48 mm Hg Arterial pH: 7.49 Plasma HCO3-: 35 mEq/L
Which of the following is most likely decreased in this man?
1)
2)
3)
4)
5)

Plasma aldosterone
Plasma potassium
Potassium excretion
Renin secretion
Sodium excretion

The correct answer is choice B.

This is a case of metabolic alkalosis most probably caused by excessive chloride loss in urine due to diuretics overdose;

Arterial PCO2: 48 mm Hg (Normal= 35-45 mm Hg)

Arterial pH: 7.49 (Normal= 7.35 - 7.45)

Plasma HCO3-: 35 mEq/L (Normal= 22 to 26 mEq/liter)

Loop diuretics act on the Na+-K+-2Cl- cotransporter in the thick ascending limb of the loop of Henle to inhibit sodium and chloride reabsorption
i.e increasing potassium (choice C) and sodium (choice E) excretion. The increase in potassium excretion leads to a decrease in plasma
potassium levels (choice B).

The decrease in plasma potassium stimulates aldosterone secretion, which raises plasma aldosterone levels (choice A).

The sodium depletion stimulates renin secretion (choice D), which in turn raises angiotensin II levels in the plasma which also stimulates
aldosterone secretion.

Loop diuretics (as Furosemide, Bumetanide, Ethacrynic acid and Torsemide) are principally used in the following indications:

edema associated with heart failure, hepatic cirrhosis, renal impairment, nephrotic syndrome

hypertension

adjunct in cerebral/pulmonary edema where rapid diuresis is required (IV injection)

they are also sometimes used in the management of severe hypercalcemia in combination with adequate rehydration

A 43-year-old woman presents with bone pain, joint pain, and fatigue. Further history reveals blood in her urine and difficulty moving her bowels.
She denies recent weight loss, poor appetite, and night sweats. She has no chronic medical conditions and does not take any medications or
use drugs. Her family history is negative for malignancy. Physical examination is negative for lymphadenopathy or hepatosplenomegaly.
Laboratory values include serum calcium 11.2 mg/dL, serum phosphorous 2.0 mg/dL, urine calcium 410 mg, and increased levels serum 1,25hydroxy Vitamin D, and parathyroid hormone.
Patients with hypercalcemia are LEAST likely to present with which of the following symptoms:
1)
2)
3)
4)
5)

constipation
renal calculi
diarrhea
abdominal pain
mental status change

The correct answer is Choice C

Hypercalcaemia is an elevated calcium level in the blood ((Normal range: 9-10.5 mg/dL). It can be an asymptomatic laboratory finding.
However, an elevated calcium level is often indicative of other diseases and a differential diagnosis should be undertaken if it persists. It can be
due to excessive skeletal calcium release, increased intestinal calcium absorption, or decreased renal calcium excretion.

EKG findings of a shortened QT interval and a widened T wave suggest hypercalcaemia and can lead to abnormal heart rhythms. Finally, peptic
ulcers may also occur. Symptoms are more common at high calcium blood values (12.0 mg/dLl). Severe hypercalcaemia (above 1516 mg/dL)
is considered a medical emergency: at these levels, coma and cardiac arrest can result. Initial therapy consists of IV fluids and diuretics
(furosemide) supplemented with bisphosphonates and calcitonin.

The general mnemonic for remembering the physical symptoms of hypercalcaemia: "groans (constipation), moans (psychotic noise), bones
(bone pain, especially if PTH is elevated), stones (kidney stones), and psychiatric overtones (including depression and confusion)." Other
symptoms can include fatigue, anorexia, nausea, vomiting, pancreatitis and increased urination.

Constipation (Choice A) is a common physical finding of hypercalcemia.

Renal calculi (Choice B) is a common physical finding of hypercalcemia.

Abdominal pain (Choice C) is a common physical finding of hypercalcemia.

Mental status change (choice D) is a common physical finding of hypercalcemia.

A 35-year-old man presents with weakness and swollen feet. He was diagnosed 8 years ago with hypertension, refused to receive any
treatment for it. He admits experiencing nausea, anorexia, weight loss and nocturia. The results of his laboratory investigations are as follows:
Hematocrit - 30%
Random blood sugar - 80 mg/dL
Sodium - 135 mEq/L
Chloride - 87 mEq/L
Potassium - 6.7 mEq/L
Bicarbonate - 12 mEq/L
BUN - 32 mg/dL
Serum uric acid - 15 mg/dL
Blood pH - 7.31
Total serum proteins - 6.6g/dL
Albumin - 3.6 g/dL
Globulin - 3.0 g/dL
Serum Calcium
-Total: -2.3 mmol/L
-Ionized: -1.2 mmol/L
Which of the following would be most appropriate as the first step in his management?
1)
2)
3)
4)

Repeat electrolytes
Restrict potassium intake
Give IV calcium gluconate
Prepare for dialysis

5) Get an EKG

The correct answer is choice D.

The patient above has untreated hypertension, and is in renal failure with his labs showing anemia, hyperkalemia, metabolic acidosis and
uremia. In hyperkalemic patients, an urgent EKG is indicated to check for cardiotoxicity. IV therapy to lower serum potassium should be
commenced immediately, and preceded by IV calcium gluconate if there is cardiotoxicity. However, in the setting of renal failure, immediate
dialysis is necessary.

Figure 1: Causes of Hyperkalemia

A 37 year old African-American woman presents to the ED with abdominal pain for the past 48 hours. She has a history of alcohol related
admissions. She describes the abdominal as a sharp pain that goes to her back. She also complains of pins and needles sensation in her hands

and feet. On exam, you notice petechiae and her deep tendon reflexes are hyperreflexive. On EKG, she has a prolonged QT interval.
Which of the following is the most likely cause of the patients hypocalcemia?
1)
2)
3)
4)
5)

Sarcoidosis
Malignancy
acute pancreatitis
Pagets disease
Milk-alkali syndrome

The correct answer is choice C

The patient has signs of acute pancreatitis complicated by hypocalcemia. Hypocalcaemia is the presence of low serum calcium levels in the
blood, usually taken as less than 2.1 mmol/L or 9 mg/dl or an ionized calcium level mm of less than 1.1 mmol/L (4.5 mg/dL). About half of all
calcium is bound to proteins such as serum albumin, but it is the unbound, or ionized, calcium that the body regulates. If a person has abnormal
levels of blood proteins, then the plasma calcium may be inaccurate. In the setting of low serum albumin (frequently seen in patients with
chronic diseases, hepatic disease or even long term hospitalization), the formula for corrected calcium is: CorrCa = Measured serum Ca + [(4.0
- measured serum albumin) * 0.8]. Thus, if the albumin is low, the measured calcium may appear low when in fact it is physiologically within
normal limits.

Physical signs of hypocalcemia can include petechia, perioral tingling and paraesthesia, tetany, Trousseau sign of latent tetany (eliciting carpal
spasm by inflating the blood pressure cuff and maintaining the cuff pressure above systolic), tendon reflexes are hyperactive, and Chvostek's
sign (tapping of the inferior portion of the zygoma will produce facial spasms). On EKG, the QT interval is often prolonged.

To correct hypocalcemia, intravenous calcium gluconate is given slowly in a period of 10 minutes, or if the hypocalcaemia is severe, calcium
chloride is given instead. Maintenance doses of both calcium and vitamin-D (often as 1,25-(OH)2-D3, i.e. calcitriol) are often necessary to
prevent further decline.

sarcoidosis (choice A) frequently causes a dysregulation of vitamin D production with an increase in extrarenal production. Specifically,
macrophages inside the granulomas convert vitamin D to its active form, resulting in elevated levels of the hormone 1,25-dihydroxyvitamin D
and symptoms of hypervitaminosis D that may include fatigue, lack of strength or energy, irritability, metallic taste, temporary memory loss or
cognitive problems.

Malignancy (choice B) is a common cause of hypercalcemia through the production of PTH related petide secreting tumors (PTHrP). PTHrP
secreting tumors are often solid tumour with metastasis (e.g. breast cancer or classically squamous cell carcinoma), solid tumour with humoral
mediation of hypercalcaemia (e.g. lung or kidney cancer, pheochromocytoma), and hematological malignancies (multiple myeloma, lymphoma,
leukemia).

Pagets disease (choice D) is a characterized by high bone-turnover rate that leads to hypercalecmia. The pathogenesis of Paget's disease is
described in 3 stages which are:osteoclastic activity, mixed osteoclastic-osteoblastic activity, and finally exhaustive (burnt out) stage. Initially,
there is a marked increase in the rate of bone resorption at localized areas caused by large and numerous osteoclasts. These localized areas of
osteolysis are seen radiologically as an advancing lytic wedge in long bones or osteoporosis circumscripta in the skull. The osteolysis is
followed by a compensatory increase in bone formation induced by osteoblasts recruited to the area. This is associated with accelerated
deposition of lamellar bone in a disorganized fashion. This intense cellular activity produces a chaotic picture of trabecular bone ("mosaic"
pattern), rather than the normal linear lamellar pattern. The resorbed bone is replaced and the marrow spaces are filled by an excess of fibrous
connective tissue with a marked increase in blood vessels, causing the bone to become hypervascular. The bone hypercellularity may then
diminish leaving a dense "pagetic bone," also known as burned-out Paget disease.

Milk-alkali syndrome (chocie E) is characterized by hypercalcemia caused by repeated ingestion of calcium and absorbable alkali (such as
calcium carbonate, or milk and sodium bicarbonate). The name "milk-alkali syndrome" derives from when patients would take in excessive
amounts of milk and antacids to control their dyspepsia.

An 75 year-old male presents with lethargy and anorexia for three days duration. He has a history of atrial fibrillation and has been rate
controlled for several years with digoxin. His history is otherwise unremarkable, and his physical exam is within normal limits. After labs are
drawn he is found to have a plasma potassium level of 6.0 mEq/L and a digoxin level of 4.0 (normal therapeutic levels are between 0.8 - 2.0). A
stat EKG shows peaked T waves and widening of the QRS complex.
Which of the following would should be administered first to treat the patients hyperkalemia?
1)
2)
3)
4)
5)

administration of calcium gluconate intravenously


administration of glucose, insulin and sodium bicarbonate intravenously
albuterol
oral administration of potassium exchange resins with sorbitol
hemodialysis with low potassium dialysate

The correct answer is choice A

Hyperkalemia refers to the condition in which the concentration of the electrolyte potassium (K+) in the blood is elevated. Extreme hyperkalemia
is a medical emergency due to the risk of potentially fatal abnormal heart rhythms (arrhythmia). Normal serum potassium levels are between 3.5
to 5.0 mEq/L; at least 95% of the body's potassium is found inside cells, with the remainder in the blood. This concentration gradient is
maintained principally by the Na+/K+ pump. Increased extracellular potassium levels result in depolarization of the membrane potentials of cells.
This depolarization opens some voltage-gated sodium channels, but not enough to generate an action potential. After a short while, the open
sodium channels inactivate and become refractory, increasing the threshold needed to generate an action potential. This leads to the
impairment of neuromuscular, cardiac, and gastrointestinal organ systems.

The renal elimination of potassium is passive (through the glomeruli), and reabsorption is active in the proximal tubule and the ascending limb of
the loop of Henle. There is active excretion of potassium in the distal tubule and the collecting duct; both are controlled by aldosterone.
Hyperkalemia develops when there is excessive production (oral intake, tissue breakdown) or ineffective elimination of potassium. Ineffective
elimination can be hormonal (in aldosterone deficiency) or due to causes in the renal parenchyma that impair excretion.

Symptoms are fairly nonspecific and generally include malaise, palpitations and muscle weakness; mild hyperventilation may indicate a
compensatory response to metabolic acidosis, which is one of the possible causes of hyperkalemia. Often, however, the problem is detected
during screening blood tests for a medical disorder, or it only comes to medical attention after complications have developed, such as cardiac
arrhythmia or sudden death. During the medical history taking, a physician will focus on kidney disease and medication use.

When arrhythmias occur, or when potassium levels exceed 6.5 mmol/l, emergency lowering of potassium levels is mandated. Several agents
are used to lower K levels. Choice depends on the degree and cause of the hyperkalemia, and other aspects of the patient's condition. Calcium
supplementation (calcium gluconate 10%, preferably through a central venous catheter as the calcium may cause phlebitis) does not lower
potassium but decreases myocardial excitability, protecting against life threatening arrhythmias. It tends to be used if there are characteristic
EKG changes.

administration of glucose, insulin and sodium bicarbonate intravenously (choice B) will lead to a shift of potassium ions into cells, secondary to
increased activity of the sodium-potassium ATPase. Its effects last a few hours, so it sometimes needs to be repeated while other measures are
taken to suppress potassium levels more permanently. Bicarbonate therapy is effective in cases of metabolic acidosis. The bicarbonate ion will
stimulate an exchange of cellular H+ for Na+, thus leading to stimulation of the sodium-potassium ATPase.

albuterol (choice C) is a 2-selective catecholamine that is administered by nebulizer (e.g. 1020 mg). This drug promotes movement of K into
cells, lowering the blood levels.

oral administration of potassium exchange resins with sorbitol (choice D) such as Kayexalate promotes the lowering of potassium through the
GI tract. The potassium is removed when the patient has a bowel movement.

hemodialysis with low potassium dialysate (chocie E) is used to treat hyperkalemia in refractory or severe cases when potassium needs to be
removed from cicrulation.

A 35-year-old woman presents to her physician complaining of headaches, increased sweating, and intermittent stabbing pain on her right side.
She has no chronic medical conditions, does not take any medication, and does not smoke or use other drugs or alcohol. Her family history is
significant for thyroid cancer on her maternal side, type unknown. On physical examination, the patient is pale and anxious appearing. Her blood
pressure is 210/130 and her heart rate is 120.
High serum calcium/low phosphorus/increased alkaline phosphatase are associated with which of the following conditions?
1)
2)
3)
4)
5)

Sarcoidosis
Malignancy
Primary hypophosphatemia
Pagets disease
Milk-alkali syndrome

The correct answer is choice B

Multiple endocrine neoplasia (MEN) type II, also known as Sipples syndrome, is characterized by medullary carcinomaof the thyroid,
pheochromocytoma, and parathyroid tumor or adenoma. MEN type I, also known as Wermers syndrome, is characterized by pancreatic,
parathyroid, and pituitary tumors. Both MEN type I and II are inherited in an autosomal-dominant fashion. In this case, the clinical history
includes several hyperadrenergic symptoms consistent with pheochromocytoma, including marked hypertension, tachycardia, pallor, and
diaphoresis. Additional symptoms of pheochromocytoma include palpitations, anxiety, and weight loss. A common manifestation of
hyperparathyroidism is hypercalcemia, which can result in kidney stones and flank pain. These findings, when coupled with a family history of
thyroid cancer, should make one suspicious of MEN type II.

Malignancy is a common cause of hypercalcemia through the production of PTH related petide secreting tumors (PTHrP). PTHrP secreting
tumors are often solid tumour with metastasis (e.g. breast cancer or classically squamous cell carcinoma), solid tumour with humoral mediation
of hypercalcaemia (e.g. lung or kidney cancer, pheochromocytoma), and hematological malignancies (multiple myeloma, lymphoma, leukemia).

Hypercalcaemia is an elevated calcium level in the blood ((Normal range: 9-10.5 mg/dL). It can be an asymptomatic laboratory finding.
However, an elevated calcium level is often indicative of other diseases and a differential diagnosis should be undertaken if it persists. It can be
due to excessive skeletal calcium release, increased intestinal calcium absorption, or decreased renal calcium excretion.

EKG findings of a shortened QT interval and a widened T wave suggest hypercalcaemia and can lead to abnormal heart rhythms. Finally, peptic
ulcers may also occur. Symptoms are more common at high calcium blood values (12.0 mg/dLl). Severe hypercalcaemia (above 1516 mg/dL)
is considered a medical emergency: at these levels, coma and cardiac arrest can result. Initial therapy consists of IV fluids and diuretics
(furosemide) supplemented with bisphosphonates and calcitonin.

The general mnemonic for remembering the physical symptoms of hypercalcaemia: "groans (constipation), moans (psychotic noise), bones
(bone pain, especially if PTH is elevated), stones (kidney stones), and psychiatric overtones (including depression and confusion)." Other
symptoms can include fatigue, anorexia, nausea, vomiting, pancreatitis and increased urination.

sarcoidosis (choice A) frequently causes a dysregulation of vitamin D production with an increase in extrarenal production. Specifically,
macrophages inside the granulomas convert vitamin D to its active form, resulting in elevated levels of the hormone 1,25-dihydroxyvitamin D
and symptoms of hypervitaminosis D that may include fatigue, lack of strength or energy, irritability, metallic taste, temporary memory loss or
cognitive problems.

Primary hypophosphatemia (choice C) is the most common cause of nonnutritional rickets. Laboratory findings include low-normal serum
calcium and hypercalciuria, moderately low serum phosphate, elevated serum alkaline phosphatase, and low serum 1,25 dihydroxy-vitamin D
levels, hyperphosphaturia, and no evidence of hyperparathyroidism. Primary hypophosphatemia is also characterized by direct excess excretion
of phosphate by the kidneys, as from primary renal dysfunction, and also the direct action of many classes of diuretics on the kidneys.
Additionally, secondary causes, including both types of hyperparathyroidism cause hyperexcretion of phosphate in the urine.

Pagets disease (choice D) is a characterized by high bone-turnover rate that leads to hypercalecmia. The pathogenesis of Paget's disease is
described in 3 stages which are:osteoclastic activity, mixed osteoclastic-osteoblastic activity, and finally exhaustive (burnt out) stage. Initially,
there is a marked increase in the rate of bone resorption at localized areas caused by large and numerous osteoclasts. These localized areas of
osteolysis are seen radiologically as an advancing lytic wedge in long bones or osteoporosis circumscripta in the skull. The osteolysis is
followed by a compensatory increase in bone formation induced by osteoblasts recruited to the area. This is associated with accelerated
deposition of lamellar bone in a disorganized fashion. This intense cellular activity produces a chaotic picture of trabecular bone ("mosaic"
pattern), rather than the normal linear lamellar pattern. The resorbed bone is replaced and the marrow spaces are filled by an excess of fibrous
connective tissue with a marked increase in blood vessels, causing the bone to become hypervascular. The bone hypercellularity may then
diminish leaving a dense "pagetic bone," also known as burned-out Paget disease. Pagets is characterized by normal serum calcium, normal
phosphorus, and very high alkaline phosphatase.

Milk-alkali syndrome (chocie E) is characterized by hypercalcemia caused by repeated ingestion of calcium and absorbable alkali (such as
calcium carbonate, or milk and sodium bicarbonate). The name "milk-alkali syndrome" derives from when patients would take in excessive
amounts of milk and antacids to control their dyspepsia. Phosphorus and alkaline phosphatase levels are normal.

A 43-year-old retired air hostess with chronic kidney disease on an aggressive hemodialysis program arrives at the emergency department
complaining of dark urine. The patient says that this is the first time that she has ever had this problem. On physical exam, there is noted
muscle weakness in all extremities.
His urinalysis shows:

blood++
protein ++
glucose neg
leukocycte esterase neg
nitrites neg

Urine microscopy is negative for RBCs and shows some epithelial cells. Blood tests are notable for CK 1000 U/L and low serum phosphate.
Complications of severe hypophosphatemia include all of the following, except:
1)
2)
3)
4)

Increased serum CPK enzyme


Respiratory muscle weakness
Intra-vascular hemolysis
Hypocalciuria

5) Cardiac dysrrhythmias

The correct answer is choice D

Hypophosphatemia is an electrolyte disturbance in which there is an abnormally low level of phosphate in the blood. The condition has many
causes, but is most commonly seen when malnourished patients (especially chronic alcoholics) are given large amounts of carbohydrates,
which creates a high phosphorus demand by cells, removing phosphate from the blood (refeeding syndrome). Because a decrease in
phosphate in the blood is sometimes associated with an increase in phosphate in the urine, the terms hypophosphatemia and "phosphaturia"
are occasionally used interchangeably.

Primary hypophosphatemia is the most common cause of nonnutritional rickets. Laboratory findings include low-normal serum calcium and
hypercalciuria, moderately low serum phosphate, elevated serum alkaline phosphatase, and low serum 1,25 dihydroxy-vitamin D levels,
hyperphosphaturia, and no evidence of hyperparathyroidism. Primary hypophosphatemia is also characterized by direct excess excretion of
phosphate by the kidneys, as from primary renal dysfunction, and also the direct action of many classes of diuretics on the kidneys. Additionally,
secondary causes, including both types of hyperparathyroidism cause hyperexcretion of phosphate in the urine.

The pathophysiology of hypophosphatemia is caused by the following three mechanisms:


Inadequate intake (often unmasked in refeeding after long-term low phosphate intake)
Increased excretion (e.g. in hyperparathyroidism)
Shift from extracellular to intracellular space (seen in treatment of diabetic ketoacidosis, refeeding, short-term increases in cellular demand (e.g.,
hungry bones syndrome) and acute respiratory alkalosis)

Major symptoms of hypophosphatemia include muscle dysfunction and weakness. This occurs in major muscles, but also may manifest as
diplopia, low cardiac output, dysphagia, and respiratory depression due to respiratory muscle weakness. Mental status changes. This may
range from irritability to gross confusion, delirium, and coma. White cell dysfunction with subsequent worsening of infections. Instability of cell
membranes due to low ATP levels leading to rhabdomyolysis with increased CPK.

Increased serum CPK enzyme (choice A) is a common finding of hypophosphatemia due to the instability of cell membranes. Low ATP levels

lead to rhabdomyolysis with increased CPK.

Respiratory muscle weakness (chocice B) is a common physical finding of hypophosphatemia.

Intra-vascular hemolysis (choice C) is a common finding of hypophosphatemia due to the instability of cell membranes from low ATP levels.

Cardiac dysrrhythmias (choice E) is a common finding of hypophosphatemia due to dysfunction and weakness of cardiac tissue from low ATP
levels that results in low cardiac output and dysrrhythmias.

A 68 year old man with uncontrolled diabetic is admitted to the ICU after falling comatose in a nursing home. He has recently underwent a
urinary bladder resection several weeks ago and takes cholestyramine for Crohn's disease. He has been having active diarrhea for the past few
days due to antibiotic induced colitis.
An increased anion gap is associated with which of the following conditions?
1)
2)
3)
4)
5)

Diarrhea
radical surgery of urinary bladder
cholestyramine
lactic acidosis
renal tubular acidosis

The correct answer is choice D

To distinguish between the main types of metabolic acidosis, a clinical tool called the anion gap is considered very useful. It is calculated by
subtracting the chloride and bicarbonate levels from the sodium.

Anion gap = ( [Na+] ) - ( [Cl-]+[HCO3-] )

As sodium is the main extracellular cation, and chloride and bicarbonate are the main anions, the result should reflect the remaining anions.
Normally, this concentration is about 8-16 mmol/l (124). An elevated anion gap (i.e. > 16 mmol/l) can indicate particular types of metabolic
acidosis, particularly certain poisons, lactate acidosis and ketoacidosis.
Caluclating this patients anion gap reveals a normal anion gap, which shortens the differential diagnosis considerably.

The most common causes of normal anion gap acidosis are GI or renal bicarbonate loss and impaired renal acid excretion. Normal anion gap
metabolic acidosis is also called hyperchloremic acidosis, because instead of reabsorbing HCO3 with Na, the kidney reabsorbs Cl. Many GI
secretions are rich in bicarbonate (eg, biliary, pancreatic, and intestinal fluids); loss from diarrhea, tube drainage, or fistulas can cause acidosis.
In ureterosigmoidostomy (insertion of ureters into the sigmoid colon after obstruction or cystectomy), the colon secretes and loses bicarbonate
in exchange for urinary Cl and absorbs urinary ammonium, which dissociates into NH3+ and H+.

A useful mnemonic that summarizes the causes of normal anion gap acidosis is DURHAM

a) Diarrhea (HCO3- and water is lost).

b) Ureteral diversion: Urine from the ureter may be diverted to the sigmoid colon due to disease (uretero-colonic fistula) or after bladder
surgery. In such an event urinary Cl- is absorbed by the colonic mucosa in exchange for HCO3-, thus increases the gastrointestinal loss
of HCO3-.

c) Renal tubular acidosis: dysfunctional renal tubular cells causes an inappropriate wastage of HCO3- and retention of Cl-.

d) Hyperalimentation

e) Acetazolamide

f) Miscellaneous conditions: They include pancreatic fistula, cholestyramine, and calcium chloride (CaCl) ingestion, all of which can
increase the gastrointestinal wastage of HCO3-.

Lactic acid results from lactic acidosis and is considered a distinct form of high anion gap metabolic acidosis. The condition typically occurs
when tissues receive too little oxygen (hypoxia or hypoperfusion) and cells are forced to metabolize glucose anaerobically. Therefore, excess
pyruvate produced is converted into lactate and released into the bloodstream. Lactic acidosis is characterized by lactate levels >5 mmol/L and
serum pH <7.35.

diarrhea (choice A) leads to a nongap acidosis from a loss of GI secretions which are rich in bicarbonate.

radical surgery of urinary bladder (choice B) can lead to a normal anion gap acidosis from ureteral diversion. Urine from the ureter may be
diverted to the sigmoid colon due to disease (uretero-colonic fistula) or after bladder surgery. subsequently, urinary Cl- is absorbed by the
colonic mucosa in exchange for HCO3- and increase the gastrointestinal loss of HCO3-.

cholestyramine (choice C) increases the gastrointestinal wastage of HCO3- leading to a normal gap acidosis.

renal tubular acidosis (choice E) leads to a nongap acidosis from dysfunctional renal tubular cells that causes an inappropriate wastage of
HCO3- and retention of Cl-.

A 47-year-old man with systolic heart failure and hepatitis C comes to the Emergency Department with shortness of breath and swelling of his
ankles. His past medical history is significant for small cell lung cancer, recent head trauma, and adrenal gland insufficiency and he is on
metformin. On physical exam, the patient is found to have 2+ pitting edema in both lower extremities. The patient has been on furosemide for
the past year. His metabolic panel is within normal limits with the exception of serum sodium of 128 mEq/L.
Which of the following is the most likely cause of hyponatremia in this patient?

1)
2)
3)
4)
5)

diabetes insipidus
furosemide
congestive heart failure
syndrome of inappropriate antidiuretic hormone (SIADH)
Adrenal gland insufficiency

The correct answer is choice C

The patients hypervolemic status (i.e. pitting edema) and hyponatremia suggests congestive heart failure as the most likely cause.
Hyponatremia is an electrolyte disturbance in which the sodium concentration in the serum is lower than normal. Sodium is the dominant
extracellular cation and cannot freely cross the cell membrane. Normal serum sodium levels are between 135-145 mEq/L. Hyponatremia is
defined as a serum level of less than 135 mEq/L and is considered severe when the serum level is below 125 mEq/L.

Hyponatremia is most often a complication of other medical illnesses in which either fluids rich in sodium are lost (diarrhea or vomiting) or
excess water accumulates in the body at a higher rate than can be excreted (for example in congestive heart failure, syndrome of inappropriate
antidiuretic hormone, SIADH, or polydipsia). Regarding sodium loss as a cause of hyponatremia, it is important to note that such losses promote
hyponatremia in only an indirect manner. In particular, hyponatremia occurring in association with sodium loss does not reflect inadequate
sodium availability as a result of the losses. Rather, the sodium loss leads to a state of volume depletion, with volume depletion serving as
signal for the release of ADH (anti-diuretic hormone). As a result of ADH-stimulated water retention, blood sodium becomes diluted and
hyponatremia results.

Symptoms of hyponatremia include nausea and vomiting, headache, confusion, lethargy, fatigue, appetite loss, restlessness and irritability,
muscle weakness, spasms, or cramps, seizures, and decreased consciousness or coma. The presence and severity of symptoms are
associated with the level of serum sodium, with the lowest levels of serum sodium associated with the more prominent and serious symptoms.

Neurological symptoms often show for extremely low levels of sodium. When sodium levels in blood become too low, excess water enters cells
and causes the cells to swell. Swelling in the brain is especially dangerous because the brain is confined by the skull and is unable to expand.
Neurological symptoms most often are due to very low serum sodium levels (usually <115 mEq/L), resulting in intracerebral osmotic fluid shifts
and brain edema. This neurological symptom complex can lead to tentorial herniation with subsequent brain stem compression and respiratory
arrest, resulting in death in the most severe cases.

The imbalance between sodium and water in blood may occur in three primary ways and is based on the patients volume status:

Hypervolemic hyponatremia, excess water dilutes the sodium concentration, causing low sodium levels. Hypervolemic hyponatremia is
commonly the result of kidney failure, heart failure or liver failure.
Euvolemic hyponatremia, normal water levels are combined with low sodium levels. This condition is commonly due to chronic health
conditions, cancer or certain medications.|

Hypovolemic hyponatremia, water and sodium levels are both low. This may occur, for example, when exercising in the heat without
replenishing fluid electrolytes or with marked blood loss.

Diabetes insipidus (choice A) is caused by excessive excretion of water from the kidneys due to inadequate production of the
hormone,vasopressin, from the pituitary gland or impaired responsiveness of the kidneys to vasopressin. It would result in a euvolemic
hypernatremia.

furosemide (choice B) would result in urinary sodium loss and lowered volume status. Patients would exhibit an elevated BUN:Creatinine ratio
and other physical signs of volume depletion such as tenting of the skin and reduced capillary refill. Diuretics such as furosemide would result in
a hypovolemic hyponatremia.

Syndrome of inappropriate antidiuretic hormone (choice D) is paraneoplastic syndrome commonly associated with small cell carcinoma of the
lungs but the hyponatremia is euvolemic. Patients would not present with physical findings of volume overload (i.e. ascites, pitting edema).

Adrenal gland insufficiency (choice E) is caused by insufficient steroid hormones (glucocorticoids and often mineralocorticoids by the adrenal
glands resulting in the loss of sodium, potassium and water. The hyponatremia that results would be a hypovolemic hyponatremia.

A 53-year-old taxi driver is seeing you as part of his regular hypertension follow up. His blood pressure is controlled on ACE inhibitors. He has
history of congestive heart failure, asthma , and a renal transplant, for which he takes digioxin, albuterol, and cyclosporine His lab values are
normal except an elevated serum potassium of 5.4 meq/L.
Drug-induced hyperkalemia can be caused by all of the following, except?

1)
2)
3)
4)
5)

digioxin toxicity
cyclosporine
heparin
albuterol
ACE inhibitors

The correct answer is choice D

Hyperkalemia refers to the condition in which the concentration of the electrolyte potassium (K+) in the blood is elevated. Extreme hyperkalemia
is a medical emergency due to the risk of potentially fatal abnormal heart rhythms (arrhythmia). Normal serum potassium levels are between 3.5
to 5.0 mEq/L; at least 95% of the body's potassium is found inside cells, with the remainder in the blood. This concentration gradient is
maintained principally by the Na+/K+ pump. Increased extracellular potassium levels result in depolarization of the membrane potentials of cells.
This depolarization opens some voltage-gated sodium channels, but not enough to generate an action potential. After a short while, the open
sodium channels inactivate and become refractory, increasing the threshold needed to generate an action potential. This leads to the
impairment of neuromuscular, cardiac, and gastrointestinal organ systems.

The renal elimination of potassium is passive (through the glomeruli), and reabsorption is active in the proximal tubule and the ascending limb of
the loop of Henle. There is active excretion of potassium in the distal tubule and the collecting duct; both are controlled by aldosterone.
Hyperkalemia develops when there is excessive production (oral intake, tissue breakdown) or ineffective elimination of potassium. Ineffective
elimination can be hormonal (in aldosterone deficiency) or due to causes in the renal parenchyma that impair excretion.

Symptoms are fairly nonspecific and generally include malaise, palpitations and muscle weakness; mild hyperventilation may indicate a
compensatory response to metabolic acidosis, which is one of the possible causes of hyperkalemia. Often, however, the problem is detected
during screening blood tests for a medical disorder, or it only comes to medical attention after complications have developed, such as cardiac
arrhythmia or sudden death. During the medical history taking, a physician will focus on kidney disease and medication use.

When arrhythmias occur, or when potassium levels exceed 6.5 mmol/l, emergency lowering of potassium levels is mandated. Several agents
are used to lower K levels. Choice depends on the degree and cause of the hyperkalemia, and other aspects of the patient's condition. Calcium
supplementation (calcium gluconate 10%, preferably through a central venous catheter as the calcium may cause phlebitis) does not lower
potassium but decreases myocardial excitability, protecting against life threatening arrhythmias. It tends to be used if there are characteristic
EKG changes.

albuterol is a 2-selective catecholamine that is administered by nebulizer (e.g. 1020 mg). This drug promotes movement of K into cells,
lowering the blood levels of potassium.

Digioxin toxicity (choice A) can lead to hyperkalemia. Digoxin normally competes with K+ ions for the same binding site on the Na+/K+ ATPase
pump

Cyclosporine (choice B) is an immunosuppressant drug widely used in post-allogeneic organ transplant to reduce the activity of the patient's
immune system, and therefore the risk of organ rejection. Cyclosporine interacts with a wide variety of other drugs and other substances
including grapefruit juice and can cause potassium retention, and possibly hyperkalemia.

Heparin (choice C) can cause aldosterone deficiency leading to ineffective removal of potassium from circulation.

ACE inhibitors (choice E) may cause hyperkalemia. Suppression of angiotensin II leads to a decrease in aldosterone levels. Since aldosterone
is responsible for increasing the excretion of potassium, ACE inhibitors ultimately cause retention of potassium.

Figure 1: Drugs causing hyperkalemia

A 57-year-old man with systolic heart failure and hepatitis C comes to the Emergency Department with shortness of breath and swelling of his
ankles. His past medical history is significant for small cell lung cancer and he is on metformin. On physical exam, the patient is found to have
2+ pitting edema in both lower extremities. The patient is started on furosemide. His metabolic panel is within normal limits with the exception of
serum sodium of 128 mEq/L.

Which of the following is the most likely cause of hyponatremia in this patient?
1)
2)
3)
4)
5)

diabetes insipidus
Conns syndrome
congestive heart failure
syndrome of inappropriate antidiuretic hormone (SIADH)
adrenal gland insufficiency

The correct answer is choice C

The patients hypervolemic status (i.e. pitting edema) and hyponatremia suggests congestive heart failure as the most likely cause.
Hyponatremia is an electrolyte disturbance in which the sodium concentration in the serum is lower than normal. Sodium is the dominant
extracellular cation and cannot freely cross the cell membrane. Normal serum sodium levels are between 135-145 mEq/L. Hyponatremia is
defined as a serum level of less than 135 mEq/L and is considered severe when the serum level is below 125 mEq/L.

Hyponatremia is most often a complication of other medical illnesses in which either fluids rich in sodium are lost (diarrhea or vomiting) or
excess water accumulates in the body at a higher rate than can be excreted (for example in congestive heart failure, syndrome of inappropriate
antidiuretic hormone, SIADH, or polydipsia). Regarding sodium loss as a cause of hyponatremia, it is important to note that such losses promote
hyponatremia in only an indirect manner. In particular, hyponatremia occurring in association with sodium loss does not reflect inadequate
sodium availability as a result of the losses. Rather, the sodium loss leads to a state of volume depletion, with volume depletion serving as
signal for the release of ADH (anti-diuretic hormone). As a result of ADH-stimulated water retention, blood sodium becomes diluted and
hyponatremia results.

Symptoms of hyponatremia include nausea and vomiting, headache, confusion, lethargy, fatigue, appetite loss, restlessness and irritability,
muscle weakness, spasms, or cramps, seizures, and decreased consciousness or coma. The presence and severity of symptoms are
associated with the level of serum sodium, with the lowest levels of serum sodium associated with the more prominent and serious symptoms.

Neurological symptoms often show for extremely low levels of sodium. When sodium levels in blood become too low, excess water enters cells
and causes the cells to swell. Swelling in the brain is especially dangerous because the brain is confined by the skull and is unable to expand.
Neurological symptoms most often are due to very low serum sodium levels (usually <115 mEq/L), resulting in intracerebral osmotic fluid shifts
and brain edema. This neurological symptom complex can lead to tentorial herniation with subsequent brain stem compression and respiratory
arrest, resulting in death in the most severe cases.

The imbalance between sodium and water in blood may occur in three primary ways and is based on the patients volume status:
Hypervolemic hyponatremia, excess water dilutes the sodium concentration, causing low sodium levels. Hypervolemic hyponatremia is
commonly the result of kidney failure, heart failure or liver failure.
Euvolemic hyponatremia, normal water levels are combined with low sodium levels. This condition is commonly due to chronic health
conditions, cancer or certain medications.
Hypovolemic hyponatremia, water and sodium levels are both low. This may occur, for example, when exercising in the heat without
replenishing fluid electrolytes or with marked blood loss.

Diabetes insipidus (choice A) is caused by excessive excretion of water from the kidneys due to inadequate production of the
hormone,vasopressin, from the pituitary gland or impaired responsiveness of the kidneys to vasopressin. It would result in a euvolemic
hypernatremia.

Conns syndrome (choice B) or primary hyperaldosteronism, is characterized by the overproduction of the mineralocorticoidhormone
aldosterone by the adrenal glands. It would cause a Hypervolemic hypernatremia.

Syndrome of inappropriate antidiuretic hormone (choice D) is paraneoplastic syndrome commonly associated with small cell carcinoma of the
lungs but the hyponatremia is euvolemic. Patients would not present with physical findings of volume overload (i.e. ascites, pitting edema).

Adrenal gland insufficiency (choice E) is caused by insufficient steroid hormones (glucocorticoids and often mineralocorticoids by the adrenal
glands resulting in the loss of sodium, potassium and water. The hyponatremia that results would be a hypovolemic hyponatremia.

An 17 year old high school student presents with the sudden onset of fever, headache, myalgias, and nausea and vomiting twelve hours ago.
He also complains of neck stiffness and photophobia. Vital signs include temperature 39.7 C (103.5 F), pulse 120, blood pressure 95/60, and
respirations 24/min. On physical examination, he is somnolent but arousable. There is pronounced nuchal rigidity. With the hips flexed, attempts
to straighten the knee result in spasm of the hamstrings. Small, 1-2mm petechial lesions are present on the trunk, legs, and soft palate. Therapy

with intravenous fluid and supplemental oxygen is begun, and lumbar puncture is performed.
Which of the following is the most likely finding on CSF analysis in this patient?
1)
2)
3)
4)
5)

Increased WBC with neutrophilic predominance, increased protein, decreased glucose


Increased protein with oligoclonal bands of IgG
Xanthrochromia and increased RBCs
Increased WBC with lymphocytic predominance, increased protein, normal glucose
Increased WBC with lymphocytic predominance, numerous RBCs, elevated protein, normal glucose

The correct answer is choice A.

This patient has bacterial meningitis, most likely caused by N.meningitides. The most likely findings on CSF analysis would include an elevated
white count with neutrophilic predominance, decreased glucose, and increased protein (answer A).

On many standardized tests, you likely would have been given exact laboratory values for a question such as this, meaning you would have had
to determine in which ways they differ from the norm. Dont be overwhelmed by CSF analysis just like with other labs, a few common-sense
facts will get you a long way.

1. The presence of PMNs in the CSF usually indicates bacterial infection, while finding lots of lymphocytes indicates viral infection just as
it would in the blood.

2. If the glucose in the CSF is low, then it stands to reason that something is using it up. In matter of fact, it is the PMNs that cause a low
glucose, but it makes an easy-to-remember mental picture to think of bacteria as gobbling up all of the glucose (and helps explain why
the glucose is normal or only slightly decreased in viral meningitis).

3. Protein is normally kept out of the CSF by the blood-brain and blood-CSF barriers, so finding an elevated protein indicates meningeal
irritation. Of course, the proteins can also be antibodies, so conditions like multiple sclerosis and Guillain-Barre may also cause elevated
protein.

One additional teaching point Meningeal signs include the presence of nuchal rigidity (resistance of the neck to passive flexion), Kernigs sign
(described in the question stem), and Brudzinskis sign (when there is resistance to passive flexion of the neck with both hips and knees flexed).
Any and all of these signs indicate meningeal irritation.

Increased protein with oligoclonal bands of IgG (answer B) is seen pathognomonic for multiple sclerosis.

Xanthrochromia and increased RBCs (answer C) are seen in subarachnoid hemorrhage. Xanthrochromia results from the breakdown of red
cells, so finding pink or yellow CSF is more likely to mean subarachnoid hemorrhage than a traumatic tap.

An increased WBC with lymphocytic predominance, increased protein, normal glucose(answer D) would be more consistent with a viral
meningitis.

An increased WBC with lymphocytic predominance and numerous RBCs (answer E) would be consistent with herpes encephalitis. Apart from a
traumatic tap or a subarachnoid hemorrhage, the other major cause for a large number of RBCs in the CSF is herpes encephalitis - which is
itself a commonly tested point.

A 22 year old male patient was operated on for acute appendicitis. During the operation, he suffered from a severe cardiac arrythmia. His
serum K+ level was found to be low (2.4 mmol/l). Despite massive infusions of K+, his hypokalemia remained unresolved.
He had previously had an episode of ileus during which he had muscle cramps in his fingers. Serum K+ levels were not measured at this time.
Vital signs were also normal except there were pathologic orthostatic changes: the pulse increased from 78 to 90 beats/min after 2 min of
standing upright. The cardiac rhythm was not exactly regular (the ECG later showed two supraventricular extra systoles/min). The chest, lung,
abdomen and skin showed no obvious abnormalities. Muscular strength was unremarkable. Physical examination was unremarkable except a
tendency for excessive thirst. Tests for diabetes mellitus were negative. Repeated urine analysis failed to detect presence of any diuretic use.
Metabolic alkalosis, hypomagnesia and hypocalciuria were also present. High concentration of Cl- was present in the urine.
What is the most probable cause of the chronic hypokalemia observed in the patient?
1) Bartters syndrome
2) Gitelman syndrome

3) Renal tubular acidosis


4) Vomiting
5) Thiazide abuse

The correct answer is choice B.

Gitelman syndrome may account for ~50% of all chronic hypokalaemia. The patients are adults with chronic benign hypokalaemia, they have no
significant health problems related to hypokalaemia. Low blood pressure, tiredness and increased fatigability are common; there will be normal
renal function, borderline hypochloraemia, metabolic alkalosis and secondary hyperaldosteronism. Diagnostically useful are the presence of
hypomagnesaemia and hypocalciuria. These features are found in most, although not all, patients with Gitelman's syndrome. The patients will
have high urinary chloride excretion rates raising suspicions of diuretic abuse, however, diuretics are absent from the urine.

The measurement of the distal fractional chloride reabsorption will demonstrate mildly abnormal ratios indicating an inability of the distal
nephron to conserve chloride completely. It has now been discovered that patients with Gitelman's syndrome have mutations of their thiazidesensitive Na/Cl co-transporter in the distal tubule of the nephron.

Bartter described a disturbance of infancy or early childhood associated with a high incidence of polyhydramnios, prematurity and retardation of
growth. The patients were profoundly hypokalaemic and had polyuria, low blood pressure, secondary hyperaldosteronism and metabolic
alkalosis. They were usually normomagnesaemic and normocalciuric. A test for distal fractional chloride reabsorption yields values of 0.50.6
(normal 0.951.0), indicating continuous renal chloride loss. Most or all of the features of Bartter's syndrome (choice A) are now explained by
several possible mutations of the genes of the chloride reabsorption mechanism of the thick ascending limb of the loop of Henle.

Renal tubular acidosis (choice C) is characterized by a hyperchloraemic metabolic acidosis and an inability to acidify the urine. Two different
types are known: proximal and distal tubular acidosis. In the former the inability to reabsorb bicarbonate in the proximal tubule is fully
compensated by distal nephron segments at a plasma bicarbonate level of 18 mmol/l. Above this level the urinary pH will not be maximally
lowered and the patients have renal sodium wasting, secondary hyperaldosteronism and mild hypokalaemia. In adults this tends to be a mild
disturbance. In children it is often part of Fanconi's syndrome and bicarbonate supplementation is required for normal growth. In distal renal
acidosis the kidney is unable to acidify the urine at any level of acidosis. Therefore, these patients may get a life-threatening, severe metabolic
acidosis. It is associated with hypokalaemia, renal sodium wasting, secondary hyperaldosteronism and usually nephrocalcinosis. Bicarbonate
and potassium supplementation are required.

Surreptitious vomiting (choice D) is a frequent cause of chronic hypokalaemia . These patients have characteristic features. They often look

emaciated and are frequently underweight. They have low normal blood pressure. There is evidence of mild renal insufficiency, hypochloraemia
and urinary chloride excretion close to zero. These findings are explained by the loss of chloride caused by vomiting. The latter also causes
metabolic alkalosis, secondary hyperaldosteronism and renal loss of potassium. Therefore, the urinary potassium concentration is usually >10
mmol/l. Owing to the metabolic alkalosis and renal sodium bicarbonate excretion the urinary sodium concentration is commonly much higher
than the urinary chloride. It is therefore not recommended that urinary sodium is measured. Surreptitious vomiting may be seen in women and
men. There are almost always neurotic or psychotic causes of the vomiting.

Diuretic abuse such as thiazide (choice E) is common in chronic hypokalaemia. Such patients sometimes give a report of excessive weight gain
or of unexplained ankle swelling, usually causing significant anxiety. Diuretic abuse occurs in persons in whom strict weight control is of major
importance to their lives. Often these patients take the diuretics only intermittently and, as a consequence, their urinary chloride concentration
may change between high (>70 mmol/l) and low (<20 mmol/l) values. They have blood pressure readings in the low normal range often together
with pathological orthostatic changes. Laboratory results usually show mild degrees of renal insufficiency, hypochloramia and metabolic
alkalosis. Urinary samples containing high concentrations of chloride will test positive for diuretics, often for thiazides or for furosemide. Except
for the presence of diuretics in the urine, diuretic abuse may be very similar to Gitelman's syndrome. Therefore, the measurement of diuretics in
urine is of great importance in these patients. It pays to use professional laboratories with established experience in the measurement of
diuretics, such as is the case in doping laboratories.

Two days ago, a 2 year old male developed a fever to 38.6 C as well as numerous episodes of emesis. Though the emesis resolved, over the
past day his mother estimates that he has had 10-12 loose, watery stools. Several other children at his daycare center have had similar
symptoms. On examination, you find a tired-appearing male who appears mild-to-moderately dehydrated. You discuss oral rehydration therapy
with the patient's mother.
Which of the following is the most appropriate additional therapy for this patient?
1)
2)
3)
4)
5)

Amoxicillin-clavulanate
Clindamycin
Loperamide
Low residue diet
Simethicone

The correct answer is choice D.

This patient has gastroenteritis, a common illness among pediatric patients. Most causes are viral, with Norwalk/norovirus, rotavirus,
adenovirus, and calicivirus being among the most common culprits.

Treatment for viral gastroenteritis is supportive; no specific therapy will lead to earlier viral eradication. The most significant issue that
complicates gastroenteritis is dehydration, which may necessitate parenteral fluids or hospitalization.

The best treatment for this patient's diarrhea is a low residue or low fiber diet. This will safely slow intestinal transit time and allow the passage
of more formed stools. A variety of such diets exist; one popular and easy to remember one is the "BRAT" diet (bananas, rice, applesauce,
toast).

Antibiotics like amoxicillin-clavulanate (choice A) or clindamycin (choice B) are ineffective for viral gastroenteritis. Even if you were concerned
that this patient had a bacterial cause of diarrhea, treatment with antibiotics may increase the likelihood of this patient developing hemolyticuremic syndrome (HUS) if the causative agent were E. coli O157:H7.

Loperamide (choice C), an anti-motility agent, does not significantly alter the natural history of diarrhea in children with gastroenteritis.
Moreover, it can lead to complications such as ileus.

Simethicone (choice E) is an antiflatulent that may help reduce pain and bloating associated with flatus. It will not alter the natural history of
gastroenteritis.

Seventy-two hours following resection of an oat cell carcinoma of the lung, a 60 year old man is found to be delirious with an altered mental
status. He is found to have a serum sodium of 125 compared to that of 140 pre-operatively. On examination his JVP is below the sternal angle,
his BP is 120/70 supine and 100/60 sitting, but he is otherwise well. His urine electrolytes are: sodium 10; potassium 10; chloride 10.
Which one of the following would be most appropriate therapy at this moment?
1)
2)
3)
4)
5)

Restrict oral and intravenous fluids


Normal saline intravenously
Hypertonic IV saline
Demeclocycline
Salt tabs

The correct answer is choice C

Hyponatremia is an electrolyte disturbance in which the sodium concentration in the serum is lower than normal. Sodium is the dominant
extracellular cation and cannot freely cross the cell membrane. Normal serum sodium levels are between 135-145 mEq/L. Hyponatremia is
defined as a serum level of less than 135 mEq/L and is considered severe when the serum level is below 125 mEq/L.

Hyponatremia is most often a complication of other medical illnesses in which either fluids rich in sodium are lost (diarrhea or vomiting) or
excess water accumulates in the body at a higher rate than can be excreted (for example in congestive heart failure, syndrome of inappropriate
antidiuretic hormone, SIADH, or polydipsia). Regarding sodium loss as a cause of hyponatremia, it is important to note that such losses promote
hyponatremia in only an indirect manner. In particular, hyponatremia occurring in association with sodium loss does not reflect inadequate
sodium availability as a result of the losses. Rather, the sodium loss leads to a state of volume depletion, with volume depletion serving as
signal for the release of ADH (anti-diuretic hormone). As a result of ADH-stimulated water retention, blood sodium becomes diluted and
hyponatremia results.

The imbalance between sodium and water in blood may occur in three primary ways and is based on the patients volume status:

* Hypervolemic hyponatremia, excess water dilutes the sodium concentration, causing low sodium levels. Hypervolemic hyponatremia is
commonly the result of kidney failure, heart failure or liver failure.

* Euvolemic hyponatremia, normal water levels are combined with low sodium levels. This condition is commonly due to chronic health
conditions, cancer or certain medications.

* Hypovolemic hyponatremia, water and sodium levels are both low. This may occur, for example, when exercising in the heat without
replenishing fluid electrolytes or with marked blood loss.

Symptoms of hyponatremia include nausea and vomiting, headache, confusion, lethargy, fatigue, appetite loss, restlessness and irritability,
muscle weakness, spasms, or cramps, seizures, and decreased consciousness or coma. The presence and severity of symptoms are
associated with the level of serum sodium, with the lowest levels of serum sodium associated with the more prominent and serious symptoms.

Neurological symptoms often show for extremely low levels of sodium. When sodium levels in blood become too low, excess water enters cells
and causes the cells to swell. Swelling in the brain is especially dangerous because the brain is confined by the skull and is unable to expand.
Neurological symptoms most often are due to very low serum sodium levels (usually <115 mEq/L), resulting in intracerebral osmotic fluid shifts
and brain edema. This neurological symptom complex can lead to tentorial herniation with subsequent brain stem compression and respiratory
arrest, resulting in death in the most severe cases. Subsequently, hypertonic saline (3% saline) must be given intravenously to treat this medical
emergency.

Restrict oral and intravenous fluids (chocie A) is an appropriate treatment for euvolemic hyponatremia when the patient is truly asymptomatic or
has only subtle symptoms. It is not the appropriate treatment to use when the a patient exhibits neurological symptoms caused by
hyponatremia.

Normal saline intravenously (choice B) is an appropriate treatment for hypovolemic hyponatremia when the patient is truly asymptomatic or has
only subtle symptoms. It is not the appropriate treatment to use when the a patient exhibits neurological symptoms caused by hyponatremia.

Demeclocycline (choice D) is widely used in the treatment of hyponatremia due to the syndrome of inappropriate antidiuretic hormone (SIADH)
when fluid restriction alone has been ineffective. Demeclocycline induces nephrogenic diabetes insipidus (dehydration due to the inability to
concentrate urine). It is not the appropriate treatment to use when the a patient exhibits neurological symptoms caused by hyponatremia.

Salt tabs (choice E) is a useful adjunct for hyponatremia but is inferior to hypertonic IV saline for acute neurological symptoms due to
hyponatremia.

A 35 year old woman was diagnosed with multiple endocrine neoplasia type 1 and a computed tomography of abdomen revealed a mass in the
right adrenal. The patient underwent transsphenoidal surgery and then total parathyroidectomy. Despite total removal of the microadenoma by
transsphenoidal surgery, there was no suppression in serum cortisol levels. So, an operation was scheduled for the adrenal tumor. Although her
calcium and phosphorus levels were normal before the surgery, four hours later she developed respiratory distress and tetany. Laboratory
studies revealed severe hyperphosphatemia and hypocalcemia.
Causes of hypocalcemia with hyperphosphatemia include all of the following, except?
1) Chronic renal failure

2)
3)
4)
5)

Hypoparathyroidism
Pseudohypoparathyroidism
Hypomagnesaemia
Acute Pancreatitis

The correct answer is choice E.

Pancreatitis is known to be a cause of hypomagnesemia and hypocalcemia due to malsorption of these electrolytes.

Hyperphosphatemia with hypocalcemia most often results because hyperphosphatemia causes low calcium levels by precipitating calcium,
decreasing vitamin D production, and interfering with parathyroid hormone-mediated bone resorption. This results in severe life-threatening
hypocalcemia. Usually the signs and symptoms of acute hyperphosphatemia are due to the effects of hypocalcemia.
Prolonged hyperphosphatemia promotes metastatic calcification, an abnormal deposition of calcium phosphate in previously healthy connective
tissues such as cardiac valves and in solid organs such as muscles. The calcium-phosphate product predicts the risk of metastatic calcification.

Excess free serum phosphorus is taken up into vascular smooth muscle via a sodium-phosphate co-transporter. The increased cellular
phosphate activates a gene, cbfa-1 that promotes calcium deposition in the vascular cell, making smooth muscle cells engage in osteogenesis.
Vascular walls become calcified and arteriosclerotic, leading to increased systolic blood pressure, widened pulse pressure, and subsequent left
ventricular hypertrophy.

Renal insufficiency (choice A) is probably the most common cause of hyperphosphatemia seen along with hypocalcemia. Endocrinopathies,
such as hypoparathyroidism (choice B), pseudohypoparathyroidism (choice C), abnormal parathyroid hormone, acromegaly and other causes of
growth hormone excess, thyrotoxicosis, glucocorticoid withdrawal or deficiency.

Hypomagnesemia (choice D) induces parathyroid hormone (PTH) resistance and also affects PTH production. Thus, hypomagnesemia also
manifests as hyperphosphatemia with hypocalcemia.
A 88 year man arrives at the ER after suffering a traumatic blow to the head from a fall in a nurshing home. The patient is nonverbal but a chart
review of past admissions reveals that he is receiving chemotherapy for small cell lung cancer and has being receiving a large amount of IV
hydration to prevent tumor lysis syndrome. He also has a history of uncontrolled diabetes and a finger stick reveals a blood glucose of 432. His

metabolic panel is within normal limits with the exception of serum sodium of 158 mEq/L.
All of the following are possible factors contributing to the patients hypernatremia except?
1)
2)
3)
4)
5)

SIADH
marked hyperglycemia
excessive IV fluids
severe water deprivation
diabetes insipidus

The correct answer is choice A

Syndrome of inappropriate antidiuretic hormone is a paraneoplastic syndrome commonly associated with small cell carcinoma of the lungs that
results in a euvolemic hyponatremia. A paraneoplastic syndrome is a disease or symptom that is the consequence of the presence of cancer in
the body, but is not due to the local presence of cancer cells. These phenomena are mediated by humoral factors (by hormones or cytokines)
excreted by tumor cells or by an immune response against the tumor. Paraneoplastic syndromes are typical among middle aged to older
patients (as are all cancers), and they most commonly present with cancers of the lung, breast, ovaries or lymphatic system (a lymphoma).
Sometimes the symptoms of paraneoplastic syndromes show even before the diagnosis of a malignancy.

Marked hyperglycemia (chocie B) can cause a hypolvolemic hypernatremia from the excessive loss of water from the urinary tract due to
osmotic diuretics of excessive blood glucose.

Excessive IV fluids (choice C) can be an iatrogenic cause of hypernatremia. Typically, the IV fluid is hypertonic fluid (a fluid with a higher
concentration of solutes than the remainder of the body) and the hypernatremia results from vigorous resuscitation .

Severe water deprivation (choice D) would cause a hypovolemic hypernatremia. Inadequate intake of water, typically in elderly or otherwise
disabled patients who are unable to take in water as their thirst dictates is common in nurshing homes and the most common cause of
hypernatremia.

Diabetes insipidus (choice E) is characterized by increased thirst (polydipsia) and increased urination (polyuria) with the inability to to
concentrate urine due to a lack of ADH. Therefore, urine specific gravity will be low (<1.006) and serum osmolality will be >250. Nephrogenic

diabetes insipidus (lack of response to ADH) can be distinguished from central (loss of ADH production) because nephrogenic DI will not
respond to DDAVP which is a ADH analog. This patient is suffering from central diabetes insipidus due to the traumatic head injury.

A 23-year-old man was rushed to the emergency room unconscious. His wife says he just returned from a trip 4 days ago and started passing
watery non-bloody stool frequently. While examining him, you observed that he had sunken eyes, lacked skin turgor, and had a tonic seizure
lasting 20 seconds. He did not pass urine or feces on himself. Random blood glucose was 90 mg/dL and his blood chemistry showed the
following result:

Sodium - 170 mEq/L


Chloride - 103 mEq/L
Potassium - 3.8 mEq/L
Bicarbonate - 15 mEq/L

Which of the following would be most appropriate in his management?


1)
2)
3)
4)
5)

Administer anticonvulsants
Administer IV normal saline
Take a sample of his stool for microscopy and culture
Administer IV bicarbonate
Pass a nasogastric tube

The correct answer is choice B.

The patient has diarrhea, features of moderate to severe dehydration and then develops a seizure. His serum electrolytes show that he has
hypernatremia and metabolic acidosis. Immediate correction of this electrolyte anomaly should be initiated with IV normal saline, which should
be switched to normal saline once he is hemodynamically stable.

Figure 1: Causes of Hypernatremia

Potrebbero piacerti anche